You are on page 1of 496
Contents PRINCIPLES OF PHARMACOLOGY | Introduction to Pharmacology 3 2 Pharmacokinesies = 5 3. Pharmacodynamics 12 4 Drug Dosing and Prescription Writing 7 M__ AUTONOMIC NERVOUS SYSTEM 5. Introduction to Autonomic Nervous System Pharmacology 23 6 Cholinergic Agonists 28 7 Cholinergic Antagonists 2... sess sereeeeseeee 37 8 Adrenergic Agonists «2.0.0. AS 9 Adrenergic Antagonists... sarcaesat 55: |_ CENTRAL NERVOUS SYSTEM 10 Introduction to Central Nervous Syseem Pharmacology «.-.....+ 65 II Anxiolytics, Hypnotics, and Sedatives : o 12 Antipsychotics .. . ” 13. Drugs Used to Treat Depression and Mania - 80 14 Anticonvulsants . 7 15 Drugs Used to Treat Parkinson s Disease and Other Movement Disorders . 95 16 Anesthetics + 102 17 CNS Stimulants -.- 21s 18 Alcohol and Orher Drugs of Abuse - 119 19 Opioid Analgesics and Antagonists 2125 1V__ CARDIOVASCULAR SYSTEM 20 Antihypertensive Drugs i 135 21 Antarehytheic Drugs z 148 22 Drugs Used to Tras Congestve Hear Failure ro 2B Diuretics a 167 24 Antianginal Drugs «=. 74 25 Anticoagulant, Fibrinlyti, and ‘cpl Drugs « 179 26 —Anhypertipidemic Drugs =... ea 27 Drugs Used to Treat Anemia... xiv Contents V__ RESPIRATORY SYSTEM 28 Drugs Used to Treat Asthma, Coughs, and Colds ..-.2...++2+++ 201 VI__ ENDOCRINE SYSTEM 29 Hypothalamic and Piuitary Hormones au 30 Thyroid and Antithyroid Drugs 225: 31 Sex Steroids and Inhibitors .. ai 32 Corticosteroids and Inhibitors... 29 33. Insulin and Oral Hypoglycemic Drugs 235 34 Drugs That Affect Calcium Homeostasis. 21 VII_ MUSCULOSKELETAL SYSTEM 35 Antiinflammatory Drugs and Acataminophon <.....s+seseee0+0 247 36 Drugs Used to Treat Gout Keanna SS 37 Autocoids and Autocoid Antagonists... 2. cseseeseeeenev es 262 VII GASTROINTESTINAL SYSTEM 38 Drugs Used to Treat Gastrointestinal Disorders 267 1x _ IMMUNE SYSTEM 39 Antineoplastic Drugs vmsindvige sedaunegpht venta TE: XX _ ANTIMICROBIAL DRUGS 40 Introduction to Antimicrobial Drugs .....0.sseseesseeeeseees 293 41 Penicilins ...-.- ssa 42 Cephalosporins and Other Cell Wal Synthesis Inhibitors 303 43 Protein Synthesis Inhibitors air 44 Quinolones and Drugs Used to Treat Urinary Tract Infections... 320 45 Folate Antagonists san 323 46 Antifungal Drugs| 328 47 Antiprotozoal Drugs... 335 48 Anthelmine Drugs. 350 49° Antiviral Drage 357 50 Drugs Used to Treat Tuberculosis and Leprosy 369 XI TOXICOLOGY 51 Toxicology .. XIL_ PHARMACOLOGY POWER REVIEW 52 Pharmacology Power Review . 397 Contents APPENDICES. A Sample Problems 8) Reconnect Daves Aare eles Creare © Comparison of Antimicrobial Spectra. . Index 451 454 461 469 Section | Principles of Pharmacology ve saignons Ya ygolosanrisld What is pharmacology? What is a drug? Name and define the four major subdivisions of | pharmacology. For each of the following ‘endings, name the classifi- cation of drug and give an ‘example: Introduction to Pharmacology ‘The study ofthe intersetion between chemicals and ling systems A drugs broadly defined as any chemical ‘agent that affects biologie systems, 1, Pharmacokineties—describes “what the body does to the drug.” Th includes topics such as absorption, distribution, metabolism, and exeretion of drugs. 2. Pharmacodynamies—describes ‘what the drug does tothe body." Specifically, it deals with the ‘biochemical and physiological effects of drugs and their mechanisms of action 8. Pharmacotherapeuties—deseribes the use of drugs forthe prevention, diagnosis, and treatment of disease Toxicology—describes the undesirable effects of therapeutic agents, poisons, and poltants on biologie systems, 4 phenothiazine-like antipsychoties chlorpromazine) volatile general anesthetics (et, halothane) antiansioty drugs (e.g, diazepam) 4 Section | Principle of Pharmacology -bital -eyeline -olol statin -zosin Should trade names be memorized for the Boards? Do I need to know every characteristic of every drug? barbiturate sedative hypnotic drugs (e, phenobarbital) local anesthoties (e.g, cocaine) penicillins (eg, nal tetraeyedin dosyeycline) ntbioties (eg Beblockers (e.g, propranolol) ACE inhibitors (e., captopeil) HMG-CoA reductase inhibitors (eg, lovastatin) postsynaptic a-receptor blockers (e.g, terazosin) ‘In the past the Boards have not tested. trade names. Its best to first learn the generic name. Trade names have been provided only for future reference. No, However, itis absolutely critical that you at least remember the classification, ‘mechanism of action, therapeutic use, and life threatening or unique adverse effects ofall ofthe major drugs. oe Define pharmacokinetics. ABSORPTION Pharmacokinetics Pharmacokinetics describes uetions of the body on drugs, inchuding the inciples of drug absorption, distribution, biotransformation (metabolism), and excretion. Define absorption. What does the rate and efficacy of absorption depend on? In what way does the pH of a drug affect its charge? Absomption isthe rate at which and extent to which a drug moves from its ste of administration, Route of administration—The stravenous route is most effective Blood flow-—Highly vascularized organs sch asthe small intestine have the sgeatest absorbing ability Surface area available—Absorption of ‘drug is direetly proportional to the surfice area availble, Solubility of a drug—The ratio of hayerophilic to ipophilie properties (partition coefficient) that a drug has will determine whether the drug ‘an permeate cell membranes, Drug-drug interactions—When given in combination, drugs can either her's enhance of inbibit 0 absorption, HA drug's acidity or alkalinity alfeets its charge, which affects absorption. Many drugs are ether weak acids oF weak bases. Acidic drugs are uncharged when protonated: HAeHt + AM Basic drugs are charged when protonated BH eB + Ht {6 Section I/ Principles of Parmacology How does charge affect a Generilly, a drug will pas through cell, drug's ability to permeate a membranes more easily iis cell membrane? uncharged. Therefore, the amount of drug absorbed depends upon its ratio of charged to uncharged species, which is determined by the ambient pH atthe site of administration and the pK, (negative log of dissociation constant) of the drug, (Figure 2-1), The fraction of administered drug that ‘gains access to its site of action ora biologic uid that allows access tothe site of action What is the bioavailability 100%—becaus all ofthe drug entors the ‘of an intravenously injected systemic circulation drug? What is the bioavailability Less than 100%—because some of the of any drug that is not intra drug may not be absorbed, or it may’ vaseularly injected? become inactivated What factors affect bio- 1, First-pass metabolism availability? 2. Allof the factors that affect absonption When pH isles than pK \When pH is greater than pk, the protonated forms the deprotonated forms HA and BH* predominate, and 8 predominate. Pk, Figure 2-1. The distribution ofa drug between its ionized and unionized form opands ‘nthe amblent pH and pK, ofthe drug For usrative purposes che drug has been 23° Signed a pK, of 65. (Redrawn trom Myeek Mj, Gertner 5B, Perper MM [Harvey RA, ‘Champe PC, ede]: Lppinet trated Review: Phormacsag, 2nd Philp, iple~ cove Raven Publishers, 1997,» 6) What factors affect bio- availability? ‘What is first-pass metabolism? What are the routes of drug administration? [Name the four types of alimentary routes of a istration and state the advantage of exch. Name the four parenteral routes of administration and state the advantage of each, Chapter 2 Pharmacolinetics 7 1. First-pass metabolism 2 Allof the factoes that affect absorption Ge, pH, blood flow, drug solubility rugedeg interactions, route of administration) Biotransformation that occurs before the mug reaches its site of ution, It most ‘commonly oceurs in the liver. (Por example, orally administered nitroglycerin s said to have a high frst pass metabolism because 90% of its Inactivated by the liver. Moxphine is another important drug tat has a high first-pass metabolism.) Alimentary Parenteral Inhalation ‘Topical Transdermal Subcutaneous 1. Oral—commonest route, Adeantazes include convenience/pationt ‘compliance and the utilization of the small intestine, which is specialized for bsomption because ofits age surface Buccal (bet sum and cheeks). Advantage: Allows direct absorption {nto the venous circulation 5, Sublingwal (ander the tongue) — Nitroglycerin is often given by th route. Advantage: Allows the dg to dri into the superior vena cava, thus bypassing hepatie first-pass ‘metabolism, 4. Rectal (suppesitory)—Usefal when the oral route is unavailable due t© ‘vomiting or loss af eonscionstess. Advantage: Approximately 50% of drug absorbed! from the rectum wll bypass the iver. 1, Intravenons direct jection into the vascular system, Advantage: Most rapid and potent mode of admini- stration, because 100% of drugenters the circulation, 8 Section | Principles of Pharmacology What category of drugs is commonly administered by inhalation? How are inhaled drugs administered? When is topical administration used? When is transdermal administration used? DISTRIBUTION 2. Intramuseular—Adleantages: Usually ‘more rapid and complete absorption than with oral administration, Minimizes hazards of intravascular ‘injection, 3. Subeutsneous—Advantages: Same as intramuscular. 4. Intrathecal—Adeantoge: In eases of acute GNS infections or spinal anesthesia, drugs can he more effective if injected direetly into the spinal subarachnoid space Pulmonary agents By machine aerosolization or vaporization Usually for treatment of localized disease (eg, psoriasis, aene, eye infections) For sustained release of a deug-—For ‘example, nicotine patches Define distribution, By what three biochemical ‘mechanisms are drugs absorbed into cells? ‘What does distribution depend upon? "The process by which a drug leaves the bloodstream and enters the interstitium ‘or the cells ofthe tissues 1. Passive diffusion—governed hy a ‘concentration gradient across a ‘membrane, which makes a drug move from an area of high concentration ta ‘one of low concentration. Itis the most common inode of drug transport 2. Transport by special carrier proteins—a form of passive diffusion that fs faellitated by a carrier protein 3, Active transport—transport agains ‘concentration gradient. The energy for this mechanism comes from dephosphorylation of adeno triphosphate Blood flow Capillary permeability The structure BIOTRANSFORMATION Why does the body biotransform drugs? ‘What are the two general sets of modifications that ‘occur in biotransformation? ‘What happens in a phase T reaction? What types of phase I reactions occur? ‘What happens in phase TI conjugation reactions? Specifically, what substrates tare added in phase IL ‘conjugation reactions? In what organ do phase Land phase Il reaetions occur? (Chapter 2/ Pharmacokinetics 9 ‘of capillaries varies depending on the corgun. For example, inthe brain the netion between cells is very tight. In we liver and spleen, the junction between endothelial cells wide, ‘which allows large molecules to pass through. Binding to plasma proteins such as albumin—This wil limit aceess to collular compartments, Drug structure—Smnall lipophilic molecules will he able to distribute to ‘more compartments than wil ange ‘The lipephilie properties of drugs that allow thera to pas through cell ‘membranes hinder their elimination ‘Therefore, drugs are modified to become more polar so that elimination can ocear more quickly ‘They are known as phase I and phase UL reactions Lipophille molecules are converted into rmore-polar molecules by introduetion of, ‘or unmacking of, a polar fumetional group, ‘Oxidation, reduction (dehydrogenation), and hydrolysis Formation of a covalent linkage between functional groups on the parent drug and another substrate Clucuronate—Quantitativey, addition of this substrate constitutes the most ‘important conjugation reaction Acetic acid Glutathione Sulfite Primarily inthe liver 10 Section |/ Principles of Pharmacology Where do these reaction ‘ocenr on « cellular level? What factors affect drug, biotransformation? Are the rates for drug bio- transformation predictable? Define first-order kinetics Deserihe zero-order kinetics Phase I reaetions aceut in the cendloplasinie reticulum, Phase I reactions occur in the cytosol Genetic differences—Bach individual has ‘avurying capacity to metabolize a drug through a given pathssay. (For example, some individuals are slow acetylators andl therefore cannot "rapidly inactivate drugs such as isoniazid, procainamide, and Iydrazine.) Inuluetion of the eytochrome P-450 system—may increase Diotransformation Inhibition ofthe cytochrome P-450 systeni—IF two drugs or compouncls are competing forthe active ste of the same enzyme, then one ofthe drugs will have a decreased rate of transformation. Disease, especially of the liver Age and gender Yes, In general, drugs will be inactivated ‘or biotransformed acconling to one of two general chemistry principles: first ‘order and zero-order kinetics. Prowess by which a const percentage ‘of substrate is metabolized per wnt time. (For example: Ten percent af a certain drug {concentration, 100 mg/L] is climinated every 2 hours: 2 hours later, the concentration will be 90 mg/l: in 4 hours it will he $1 mg/dL; and so.on.) The igher the concentration of drug, the ‘ereuter the absolute amount of drag biotransformed or exceeted per unit of time. Process by which a constant amount of ddrugis metabolized per unit of time regardless ofthe dg concentration. (For ‘example: Ifa drug concentration is 100 mga. and the body can remove 5 mg/dL. every hour, then I hous later the concentration will be 95 mgs 2 hours (Chapeor 2 Pharmacokinetics later it will he 90 mgd; and soon.) Alcohol is metabolized according to zero order kinetics EXCRETION What is exeretion? ‘The process by which a drug or metabolite is removed from the body What is the difference Excretion is the removal of a drug from between excretion and the body secretion? Soeretion ocours when the drug is actively transported from one eompartinent into another. (For example: Drugs are secreted ita the renal tubule from the medullary capillaries.) ‘What are the major routes Renal—urine is one ofthe most common of exeretion? routes of elimination Fecal Respiration—primarily for anesthetic ‘gases and vapors Breast mille skin 3 Pharmacodynamics Define pharmacodynamics, Pha ‘of «drug on the bexly, and includes the principles of receptor interactions, ‘mechanisins of therapeutic and tosie action, and dose-response relationships macodynamics describes the aetions How is pharmacodynamics ‘The pharmacokinetic processes of related to pharmacokinetics? absorption, distribution biotransformation, and excretion determine how quickly and to what extent ‘adnig wil appear ata target sit, Pharmacodynamics concepts explain the pharmacological effects of drags and thesr ‘mechanism of action (Figure 3-1). RECEPTOR INTERACTIONS What is a receptor? ‘A macromolecule typically made of proteins that interacts with either an endogenous ligand ora drug to mediate a pharmacologic or physiologic effect What are the two main 1. Ligand binding Funetions of receptors? 2. Activation of an effector system nessage propagation) What is an effector? Effectorstranschuce drug-receptor intentetions into cellular effects. There are four sypes of we mechanisms Iknonen effector 1. Transmembrane—Soine ligands such as insulin bind to receptors that have both an extracellular and intracellular component. Binding of the extracellular component stimulates the intracellular component, which is coupled toan enzyme, for example tyrosine kinase Ligand-gated ion channels Drugs Dind to these receptors, which then Chapter 3 Pharmacodynamics 13 Dose of drug administered ABSORPTION DISTRIBUTION Pesinece: rig concentration [-————-»{ Drug in tissues kinetics In systomic circulation -——_ ot distribution ———__euiminan Sap excreted ‘Drug metabolizes Pharmaco- Ciinical response Srenie Toxicty Efficacy Figure 3-1. The relationship between dote and effect can be separated inc pharmaco- lenede (dote-concentration) and pharmacodyramic (concenraton-ffect) component ‘Concentration provides the ink between pharmacokinetes and phirmacodynamss and is the focus of the target concentrauon approach to rational dosing. Te three primary processes of pharmacokinetics are absorption, dstrbution, and elimination, (Redrawn {rom Katzung 8G: Bosc and incl Pharmacaagy, Th ed. Searford, CT. Appleton & Lange, 1998, p35) alter the conductance of fons through the cell membrane channels. Examples of liganc-gated ion channel drugs ure benzodiazepines and acetylcholine 3, Intracellular—Thyroid and steroid hormones bind to nuclear receptors to form complexes that interact with DNA, which causes changes in gene expression. 4, Second messenger system—Drugs bind to receptors that activate second messenger systems involving G proteins (Figure 3-2), 14 Section 1 Principles of Pharmacology Peston "SAE ang Geta csr mzy™MeS — Giutamate R f-__~- 7. ae ae erent 6G protein-couple BHT, serotonin RS Pree cou Catayis Multsubunit Ligene-Gated cr aor Tens- fan chennela mentrane ‘Rom, Catac Activities «Proteins ——_—tectors Tyeine aces Growth acto recopters Pogulated by «subunits Cytoptesm —_Newrtopi actor receptors # Ace oylaae Tyrosine prospatases vio Sarmerrveonin iases oe eens TBFB-receptor 4 Adenylyl cyclase, cuanyy eytee ti curents AN receptor 4 cet" cont Guanytn receptor { Phosphotpase CB { Navn" exchange Mucloue ™ +cGMP_ Regulation o | “phosphodisterase tanscrpton a Steice ®) Retinoids “Thyroid hormone Figure 3-2. Chssifcaion of physologal receptors and thet relonship to seni pathway. (Redraw from Hardman JG, Limbird LE fede: eodmon and Glima's Th Phar ‘macsepeal Boss of Terapeutes, Rh ed, New York, MeGraw-Hil, 1996, p 32. Used with permission of The McGraw-Hill Companies) ‘What are second messenger Second messenger systems allow signals systems? from cell surface receptors to he converted and amplified into acellular response, What are the three best: 1, Cyclic adenosine monophosphate known second messenger (CAMP)—produced by adenylate systems, and which enzyme eyehase produces cach of them? 2 Cyclic guanosine monophosphate (cGMP) —produced by guanylate cyclase 3. Inositol triphosphate (IP,)—produced by phospholipase C Chapter 3/ Pharmacodynamics 15 MECHANISMS OF THERAPEUTIC AND TOXIC ACTION What is an agonist? ‘A drug that binds to and activates receptors What is a full agonist? A drug that, when hound to a receptor ppdces 100% of the maxin biologie response 1 possible Danugs that produce less than 100% of the ‘maximum possible biologic response no matter how high their concentration What are antagonists? Drugs that bind to reeoptors or oth drugs and dafubit a biologie response What does a competi It binds reversibly to tho same active site antagonist do? of an enzyme as an agonist. ration of the How By increasing the cone antagonist be overcome? drug (agonist). The maximum efficacy of the drug will nt change in the presence of a competitive antigo. What does a noncompetitive It bind irreversibly to a dill antagonist do? the enzyme than the antagonist Noncompetitive agonists cannot be ‘overcome by inereasing concentrations of K siteon the drug How will the maximum Masiimun efficacy will be reduced in th efficacy of a drug be presence of a noncompetitive antagonist affected by such noncompetitive antagonist (Figure 3-3), DOSE-RESPONSE RELATIONSHIPS What is the difference Efficacy isthe ability to produce a between effieney and biologie effect. Potency is related to the poteney? amount of drug necessary to cause a biologie effoot Give an example of eMfeacy. — Iftwo drugs, drug A and drug B, are both claimed to rechice patient’ heart rate by 25%, then they both have the same ff Give an example of poteney. Only 1 mg of drug A needs to be given to 16 Section | Principles of Pharmacology rug wih nor ompestve atagonist Biological ettect Drug concentration Cp for drug alone or EC lor drug in presence in presence of non- of partial agonist ‘compeltive antagonist E059 for drug in presence of partial agonist Figure 3-3. Bec of drug anegonits and parts agonist. ECyy = drug dose tha shows ‘50% of maximal response (Redrawn from Mycek MY, Gertner $8, Perper MM Harvey RA, ‘Champe PC, ed}: Lppicet' trated Review: Pharmacsogy, 2nd «4 Phila, opine cote Raven Publishers, 1997, p22) achieve a reduetion in heart rate, whoreas 10 mg of drug B are needed. Therefor can be inferred that drag 4 is more potent. What is K,? The concentration of drug yielding 50% occupancy ofthe reeeptar(dissceiation constant) What is ECs? ‘The drug concentration that produces 50% ofthe maximum possible response in 1 graded dose-response curve (see Figure 3-3), 4 DRUG DOSING Drug Dosing and Prescription Writing What three factors are involved in determi patie gan fe drug dose for a What is volume of distri- bution (Wa)? How is Vj calculated? What is the significance of a large V,?, What is a maintenance dose? What is the equation for calculating a maintenance dose? What is important to remember in performing this ealeulatios What is « loading dose? 1. Type of infection or disease 2 Patient variables (e.g, weight, liver oF kidney disease 4, Plasma concentration needed to achieve efficacy ‘The apparent volume into which a drag is able to lstibute total drug in the body + plasma ation ofthe drug Based on the equation presented above, a large Vy signifies that most ofthe drug is being sequestered in some ongan or ‘compartment. A dose of drug given to achieve a therapeutic plasma concentration over extended period of time Maintenance dose = clearance X desired plasma concentration ‘You must be absolutely certain that the In some clinical situations the desired plasma concentration ofa drug must be achieved rapidly, In these cases a sing loading dose is injected, followed routine maintenance dose, 1B Section 1/ Principles of Pharmacology ‘What is the equation for Loading dose calculating a loading dose? concentration X desired plasina Define peak and trough ‘These are maximum and tina concentrations. phism concentrations, respectively, whieh are observed during dosing since What variable affects these They will luctuate around the steal ‘concentrations? state plasma concentration (C,). ‘The point at which the rate of drug plasma concentration? availabil tothe rate of drug climination How does frequency of Tell not change. dosing affect the steady- state concentration? What factors will dosing Using smaller doses more frequent will frequency affoet? help minimize swings in drug concentration (Le, maxinnm and ‘minim plasma concenteations), See Figure 4-1 How many half-lives are Approsimately 4% half-ives. At3.3% required fo reach steady the half-life of the drug will euch 90% of state eoneentration? its effective hal Iie, What is clearance? Clearance is defined as the volume of plasma cleared of drug per unit of time What isan excretion rate? The rate at which a drug is eliminated from the body, which is measured by clearance * plasma concentration What isa therapeutic index? The ratio ofa drug's tonic dose to its therapeutic dose. A safe drug will have a high therapeutic index. See Appendix A for sumple problems illustrating these ‘concepts PRESCRIPTION WRITING Define the following abbreviations: 4 every home ahs every light (Chapter 4 / Drug Dosing and Prescripson Writing 19 Injection of Injection ot a4 20 orang Got drag 08 dally twice dally ‘Amount of drug in body (arbitrary units) = Rapid injocton of ctug Figure 4-1. Pradcted plasma concentration variasons af a drug given by infusion (8), twice cally injection (8), oF once daily ijecuon (C)- Model assumes rapd msg ina sn {de Body compartment and 2, of 12 hours (Redrawn from Mycsk MI, Gerner SB, Per- [per MM [Harvey RA, Champe PC, es) Lippincots Mustoted Reviews: Phormocoy, 2nd ed Phisdelphia, ipincote-Raven Publishers, 1997, p 20) wt everyday bid twice a dy ta three times a day sia four times day oo everynight at bedtime stat immediately te at mealtime he atnight ee after meal time 20 Section|/ Principles of Pharmacology po only st drops pm as needed Ps quantity sufficient (Ge, the pharmacist wil dispense the appropriate mumber of pills) How isa standard See Figure 4-2 preseription written? “The ist line contains the drug nae and dose: Tass 40 mg “The second line contains the directions for use Sige tab po bud “Tis ne states, “take one tablet by mouth twice a day, ‘The tied line contains the numberof tables to be dispensed: as ori Hosp 1is2 Cotas Ae. Aaa NewYork (zig eiosan0 R asx ¥0mg. HSA Figure 4-2. A sample prescription. section Il Autonomic Nervous System aoyial 2imononA i Opii388 maaexe 5 Name the to branches of the human nervous system. What are the two subdlivisions of the peripheral nervous system? What isthe autonomic nervous system? ‘Why is this system important? What are the two major subdivisions of the ANS? What are the anatomic differences between these two systems? Introduction to Autonomic Nervous System Pharmacology 1. Central nervous system 2. Peripheral nervous system 1. Somatic nervous stem, innervates skeletal musele 2, Autonomic nervous system (ANS) A collection of nucle, cell bodies, nerves, szmnglia, and plexuses that provides afferent and efferent innervation to smooth muscle and visceral organs of the body [ANS regulates fimotions that are not ‘under conscious control, such as blood pressure, heart rate, and intestinal notility,(% Also, ANS drugs have traditionally boon a favorite topie of USMLE examiners.) 1. Sympathetie nervons system 2. Parasympathetie nervous system ‘The sympathetic nervous system orig he thoracolumbar portion of the spinal cord, The preganglionic neurons are short and ‘usually synapse somewhere in the paravertebral ganglia (sympathetic hain). The portganglionte neurons are Tong and terminate at the visceral organs, ‘The parasympathetic nervous system “originates from cranial nerve nce IIL, VIL, 1X, and X, as well a the third 2 TABLE 5-1. Automatic Nervous System: Sympathetic vs Parisympathetic Responses Symputhetc Pansympathetic Effector Organs! Receptor Response Receptor Response Bye Radial muscle (iris) Contention (yaa) - Gircular muscle irs) = Ms Contraction (oss) Ciliary muscle 2“ Relanation Mj Contraction (accommodation) Heart SAnode a THR M, LHR AV node B, ——Teonduction velocity and automaticity Mz Sconduetion velocity Contractility B, Taree of contraction atria de ventricles) ML contractility (atria) Ly Bronchial muscle , ——_Relaaton (bronchoiation) My © Contestion (bonchostiion Bloc vessels Most fencept skeletal muscle) ay, ——-Constetion te Skeletal misele B, ‘Relaxation Sf G1 (Stomach nd Tntestine) Sphincter ct, Constretion (retention) M, —_Relaation (defecation) Motility and tone Ps M, motility and tone ” ones uatels sree atoUsany | cu Urinary sphincter Blader wall Uters, pregnant Uterus, nonpregnant Penis seminal vesicles Seeretory glands Sweat Intestinal Bronchial Lacrimal Metabolism Adcenal medulla Kidney Skeletal muscle Pancreas (beta cells) Fateells onstrcton Relation (retention) Contraction; resation Kelaxation Ejaculation Localized seeretion Inhibition 1 secretion (moderate) Secreto of tetanine renin release lycogendlyis,T contract: insulin release Lipohais Relaxation Conteuction Erection Generalized secretion 7 seeretion 7 sceretion Profuse secretion “The panasynpatbetis ystem controls most onqns except blood vessels, which are regulated by the nape nervous tem N= notin: nusearine receptors (Adapted from Gala G, Hann CL, Heuson WH The Pharmacology Companion. Ana Arbor, MI, Alert & Oriented Pub Aaojazeuseug wnsls snonsan awou0Iny o YoR>NpLIM § 42 st 26 Section Il / Autonome Nervous System What are the funetions of the sympathetic nervous system? With what major receptors does the sympathet nervous system work? What are the actions of the parasympathetic system? What receptors does the parasympathetic system set upon? ‘How are the parasympathetic and sympathetic systems related? neurotransmitters in the ANS? and fourth sacral spinal roots (eranionicral origins). The preginglionie neurons take a long path and synapse onto short postganglionic neurons in or near the target organ ‘The sympathetic nervous system is normally active, even at res; however, it ‘assumes a dominant role when the body becomes stressed in some way. For example, fyou sense danger, your heart rate increases, blood pressure rises, eyes dilate, blood sugar rises, bronchioles expand, and blood How shifts from the ‘The parasympathetic nervous system is predominant under tranquil conditions. It slows heart rate, lowers blood pressure, increases intestinal ativity, constriets the pupils, and empties the urinary bladder. coe stress, when itis contvolled by the sympathetic ste LL Acetylcholine — cholinergic transmission ‘Chapeer 5 / Introduccion zo Autonomic Nervous System Pharmacology 27 Which ion is required for the release of these neurotrans: rmitters from thelr storage vesicles? How do autonomie drugs function? 2, Norepinephrine—adrenengic tyansinission “The calcium fon (Ca) is required for the release of most neurotranmitters from their storage vesicles, ANS drugs achieve their effects by acting ‘as either agonists or antagonists at cholinergic and adrenergic receptors. The following four chapters discuss each of these drug clases in greater detall What are cholinergic agonists? What are the two major families of cholinergic Where in the body are cholinergic receptors found? What types of cholinergic Cholinergic Agonists Cholinengle agonists are drugs that mimic or potentiate the actions of acetylcholine 11, Musearinie—This receptor family feared its name because it was first ‘identified using musexrine, an alkaloid found in certain poisonous mushrooms 2 Nicotinic ‘There are several different subtypes of :muscatinie receptors, namely, My £0 Ms ‘They are found in ganglia, smooth ‘muscle, myocardium, secretory glands and the CNS. (® For the USMLE, not necessary to memorize which subtype of muscarinic receptors a drug will act upon.) 1 Neuronal nicotinic (Ny), located in autonomic ganglia 2. Muscular nicotinic (Ny), located in the neuromuscular junction Proganglionie fers ofthe antonomic anglia Preganglionie fers that terminate inthe ‘adrenal redalla Postgungoni bers ofthe parasympathetic system Vohntary muscles ofthe somatic system cs Sweat glands innervated hy post ganglionic syrnpathetic nervous system See Figure 6-1 Cholinergic agonists ean be divided into ‘10 major groups 1. Direct-acting agonists chemically bind with and activate muscarinie and nicotinic receptors in the body Chapter 6/ Cholinerse Agoniscs 29 Autonomic Nervous System Some Nervous Systm Srgpiteematen — Smostte | anmeanne oat Figure 6-1, Sizes of action of ehoinergie agonists nthe autonomic and somatic nervous systems. (Redravm from Myeek M), Gerter SB, Perper MM [Harvey RA, Champe PC. ds): Uippincots Moted Renews: Phorocaagy, 2nd ed. Phiselptia,Lppincoe-Raven Publishers, 1997, p36) 2. Indireetacting agonists inhibit the enzyme acetylcholinesterase and therefore increase the concentration of acetylcholine within the synapse. DIRECT-ACTING AGONISTS Give six examples of direct- 1. Acetyleholine—prototype ng ago Bethanechal Carbachal a 4. Pilocurpine 5 6 Methacholine Nicotine (discussed in Chapter 17— CNS Stimulants) ACETYLCHOLINE What are the physiologic Acetylcholine affects almost every systema actions of acetylcholine? within the body: cardiovascular system—It decreases hhoutt rate, contractility. andl blood pressure 30 Section I Autonemie Nervous System What receptors does acetylchaline activate? What are the clinical indications? What are the adverse reuctions? BETHANECHOL (Urecholine) What type of chemical compound is bethanechol? Gastrointestinal system —Tt increases motility ofthe gastrointestinal tract and bladder, Pulmonary system—Itimereases secretions ofthe bronchioles ‘The eye—It causes constriction of the pupillary sphincter musele, which ceuises miosis and accommodation, Peripheral nerwous system —Tt causes contraction of skeletal muscle Central nervous system—It affects neurotransmnission, Endocrine system—It eauses release of epinephrine from the adrenal medulla (ia nicotinic receptor), and it stimulates sweat gland secretions Both muscarinic and nicotinic Acetylcholine is used to achieve miosis during ophthalmic surgery. In general, it is rarely used because it has widespread effects and is so rapidly hydrolyzed by acetylcholinesterae, The adverse effects result from excessive generalized cholinergic stimulation. They include: Diarrhea and decreased blood pressure Urination Miosis Bronchoconstriction Excitation of skeletal muscle Lacrimation Salivation and sweating DUMBELS. a NOTE: These adverse effects are typleal ofall direct and indirect cholinergic agonists, not just acetylcholine. A carhamie acid ester What receptors does it work on? What are its therapeutic uses? What are the adverse effects of bethanechol administea- ti Chapter 6 Cholinerge Agonists 31 Bethanechal works primarily on -musearinie receptors, but It also ‘mild nicotine properties. Bethanechol increases intestinal motility, especially after surgery. Because this drug also stimulates the detrusor muscle ofthe Dad. i is also used to treat urinary retention. BBB—Betlnnechol stimulates the Bladder and Bowel the adverse effects are those that result from generalized cholinergie stimulation (see above), ‘CARBACHOL What type of compound is carbamic acid ester sila to carbachol? bethanechol State its elin What receptors docs carbachol work on? What are its adverse effects? PILOCARPINE (Plocar) What type of compound is pilocarpine? ‘What are pilocarpine’s physiologie actions? Isit cleaved by acetylcholin- esterase? ‘This drag is rarely used in the einies, but sean be used for glaucoma and to stimulate miosis daring ophthalmic surgery Both muscarinic andl lotinie receptors “Those that result from excessive genenlized cholinergie stimulation An alkaloid Cases miosis ciliary muscle Decreases heart rate contraction ofthe Cases bronchial smooth musele contraction Increases secretions from salivary lacrimal, and sweat glands No, the drug is unaffected by this 32. Section Autonomic Nervous System State the clinical use, Pilocarpine is extremely good for stimulating miosis and opening the trabecular meshwork around the eanal of Schlemm. Therefore, pilocarpine ean be used forthe treatment of glaicoms What receptors does this Primarily musearinie receptors drug work on? What are the adverse effects? Unlike the other directa previously discussed, pilocarpine is able: toont ONS dlisturbanees such as hallucinations and convulsions, along with generalized cholinengie stimulation, ig agonists the brain and cause METHACHOLINE What is methacholine used Diagnosis of esthina and bronchial for? hyperreactivity What receptors does it Musearinie receptors inate? ‘What are the adverse effets? Generalized cholineryie stimulation INDIRECT-ACTING AGONISTS: ive six examples of indircet- Tsoflurophate acting cholinergic agonists. 1 2. Echothiophate 3. Pacathion 4. Edrophoninsn 5. Physostigmine 6. Neostigmine How do they work? By inhibiting the enzyme acetylcholinesterase, which is responsible for the hydrolysis of aetylcholine, Neuronal response to acetyl therefore enhanced. Which indireet-acting choli ‘Only the organophosphates ceric agonists have the (isoflurophate, echothiophate, and ability to irreversibly inhibit parathion) irrversbly hibit avetylcholinesterase acetylel esteras Why are physostigmine, neostigmine, edrophonium, and pyridostigani considered to be reversible? Chapter 6 /Cholinarge Agonists 33 ‘ORGANOPHOSPHATES (ISOFLUROPHATE, ECHOTHIOPHATE, PARATHION) Deseribe the mechanism of action, Is it at all possible to reverse the effects of organophos- phates? What were these drugs used for in the past? What are these drugs used for today? PHYSOSTIGMINE When is physostigmine administered? ‘Organophosphates bind covalently to acetylcholinesterase and can permanently inactivate the enzyme. The effects of ‘organophosphates can last as long asa ‘week, which is approximately the time needed to synthesize a now molecule of acetylcholinesterase In mest cases, no, However, if pralidoxime (a cholinesterase reaetivator) Is given before the orgunophosphate binds to acetyleholinesterase and loses ‘one its allyl groups (a process called aging), then it may be possible for pralidexime to remove the ‘organophosphate from racetylcholinesterase (Figure 6-2) COrganophosphates were used in wars as nerve gases, They produce an immense stimulation at cholinoreceptors throughout the hods, causing respiratory smisele paralysis and convulsions, Isoflurophate and echothiophate are used becasionally for glaucoma and accommodative esotropia. Atropine is used, along with gastric lavage ‘and charcoal cessive cholinergic stimulation For glaucoma—seeond-choice drug after pilocarpine For overdoses of atropine, phenothiazines, and tricyclic antidepressants For intestinal and bowel atony For accommodative esotropia (rarely) 34 Section l/ Autonomic Nervous System Phosphorylation of Enzyme + Enzyme inactivated * Pralidaxime (PAM) can remove the inhibitor ° C3H,0-P-064H, teoturophate o-H eet at f ae oee oe Acelylcholinesteraso inactive) i ao cee (irreversibly inactive) e Low ie Figure 6-2. Covalent modifieation of acetycholinesterase by izafurophate. (Redrawn ‘rom Mycck Mi, Geancr SB, Perper MM (Harvey RA, Champe PC, ede): Uppinats Mus- ‘voted Reviews: Phrmacaloy, 2nd ed, Philadephia, Lppincott-Raven Publishers, 1997, 43) ‘Can physostigmine enter the CNS? State the adverse effects. NEOSTIGMINE (Prostigmin) Does this drug enter the cNs? Deserihe the therapeutic What is the duration of action? What are the adverse effects? Chapter 6 / Cholinergic Agonists 35 Yes, hecuuse it sa tertianyamine Convulsions ‘Muscle paralysis secondary to ‘overstiaulati Cataracts Generalized excessive cholinergic stinmulation No, because it sa polar quaternary carbamate ‘Treatment of myasthenia gravis “Treatment of urinary retention and panilytic ileus Antidote for nondepolarinzing neuromuscular blockade such as with tubocurarine Usually 2 to 4 hours Excessive cholinergic stimulation EDROPHONIUM (Enon) ‘What is its clinical use? ‘What are the adverse effects? PYRIDOSTIGMINE (Mestinon) Edrophonium is similar to neostigmine except that tis used in the diagnosis of ‘myasthenia gravis. It is not useful for maintenance therapy because ofits short duration of action (approximately 5 to 15 minutes). Edrophonium is also used to differentiate myasthenia gravis from cholinergie erisis. Both conditions can result in muscle weakness: however. ‘administration of edrophoniui helps myasthenia but worsens cholinengi criss Excessive cholinergic stimulation What is pyridostigmine’s duration of action? Very long usually 3 to 6 hours 36 Section I /Autonamie Nervous Syste What is the elinieal use? Because ofits long duration of action, pyridostigmine, like neostigmine, ean be used for long-term treatment of myasthenia gravis. What are the adverse effects? Excessive cholinergic stimulation tas Cholinergic Antagonists What are cholinergic Drugs that bind to cholinergic receptors antagonists? {muscarinie and/or nicotinic), but do not ‘rigger the usual intracellular response Name three subelasses of | L. Muscarinic blockers cholinergic antagonists. 2, Neuromuscalar blocking agents— {inhibit the efferent impulses to skeletal muscle via the nicotinic muscle receptor (Ny) 3. Ganglionic blockers—inbibit the nicotinic neuronal receptor (Ny) of Doth parasympathetic and sympathetic ganglia MUSCARINIC ANTAGONISTS. 1, Ateopi 2, Seopolamine 3, Homatropine 4. Cyelopentolate 5 6 (prototype) ‘Tropieamide Pirenzepine Are there other drugs that —-Yes—these include the anti-Parkinson's exhibit antimuscarinic drugs (eg, benetropine), the anti- properties? depressants (eg, Thomazine), antihistamines (e.g, diphenhydramine), and anti-asthmatics (e.g. ipratropium), ‘which are discussed further in later chapters. ATROPINE ‘To what family of compounds Atropine comes from the plant Atropa does atropine belong? belladonna and is known as belladonna alkaloid a7 38 Section I/ Autonomic Nervous System What is the significance of the plant's name? What is atropine’s mechan- ism of action? What agent can be used to. counteract the effeets of atropine? Does this drug eross the blood-brain barrier? What are the pharmacologic actions of atropine? List the therapeutic uses of atropine. Belladonna in Latin means pretty Indy. During the Roman era the plant was used to dilate women's pupils, whieh was ‘considered to be attractive, Tt causes reversible, nonselective blockade of musearinie receptors. High concentrations of acetylcholine oF aan equivalent muscarinic agonist No. Atropine does not readily cross the Blood -beain barrier, (ONS. At toxic doses can cause restless- ness, hallucinations, and delusions Cardiovascular system—At low doses, atropine reduces heart rate through central stimulation of the ie receptors of the heart and thns induees tachycardia Gastrointestinal system—Reduces salivary gland secretion and GI motility Pulmonary system—Reduces bronchial seeretions and stimulates bronchodilation Urinary system—Blocks muscarinic eceptors in the bladder wall, which results in bladder wall relaxation Eye—Causes paralysis ofthe sphincter rmuselo ofthe iris and elliary musele of the lens, sulting in mydriasis and cyeloplesia, Mydrlass = dilation a8) Sweat glands—Suppresses ‘sweating, espectilly in children You will more readily remernber the actions of atropine if you recognize that blocked cholinergic receptors result in an unopposed sympathetic response Bradyeardia Myuriasis and eycloplegia—beneficial When is the use of atropine to effect mydriasis and excloplegia contraindicated? How long is atropine’s duration of action? How is atropine absorbed and exereted? What are the toxic effects of this drug? SCOPOLAMINE ‘What is the classification of scopolamine? ts mechanism of How is scopolamine used therapeutically? How does this drug, from atropine? (Chapter 7/ Cholinerge Antagonists 39 ‘when a thorough fundus examination ‘or an accurate refraction is required Gastrointestinal and bladder spasms COrganophosphate poisoning Do not dilate the eyes ofa patient who has narow-angle glaucoma, becase this Approsimately 4 hours, except whem its placed in the eye, where it usually lasts shout 14 days Iis well absorbed from the gastrointestinal system and conjunetival membrane. Its excreted through both hepatic metabolism and renal filtration Tosie Effect: Mnemonic: Dry mouth Dry as a bone" Inhibition of sweating, “Hot asa hare especially in young, hile weardiaand “Red asa beet” ‘vasodilation | Blucring of vision ‘Bind as a at” Hallucinations and "Mad asa hatter deliriuen & Like atropine, this drug is belladonna allloid Nonselective competitive blockade of muscarinic receptors Prevention of motion sickness—"Totion for mation” thas a longer duration of action and ore potent CNS effects Tis offen given transdermal. 40. Secon t/ Autonomic Nervous System Are there any adverse ‘Yes—similar to those of atropine: HOMATROPINE, CYCLOPENTOLATE (Cyclogy) AND TROPICAMIDE (Mydriacy) What are these drugs used In ophthalmology, they ate given topically for? for myadrasis and eyelopleysa What are the adverse effects? Similar to those for atop milder PIRENZEPINE What A selective M, muscarinic inhibitor How is this drug used? For treating gastric uleers What are the adverse effects? Similar to those for atropine NEUROMUSCULAR BLOCKING AGENTS. [Name the two major sub- 1. Nondepolariaing blocking agents divisions of neuromuscular 2. Depolarizing blocking agents agents. NONDEPOLARIZING BLOCKING AGENTS Name four nondepolarizing 1, Tubocurarine—prototype agents. 2. Pancuronivmn—longer duration of ‘tion than tuboetrarine 3. Atracurium 4 Vecrsomium ‘What istheir mechanism of These drugs competitively block ‘action? cholinergic transmission at the nicotinic receptors by preventing the binding of acetylcholine to its receptor What isthe therapeutic use ‘They are used as adjavant drugs for of these agents? ‘anesthesia—they promote muscle relasation, Are all muscles equally No. The muscles af the eye and face are affected? affected first, whereas the respiratory muscles are affected last Chapter 7 Cholinergic Antagonists 41 What is the route of All neuromuseuiar junction blockers must sdministration? be given IV because oral absorption is poor What are the adverse effects? Bronchoconstriction and hypotension, ‘caused by histamine release What can be used to counter Because neuromisclar junction blocke act the effects of these drugs? are competitive inhibitors, the ‘can be reversed with edrophonium or neostigmine. actions DEPOLARIZING NEUROMUSCULAR BLOCKING AGENTS Name the only depolarizing Snocinylcholine neuromuscular blocking agent used in the United States. What is this drug’s mechan jam of action? Phase 1—Suecinylcholine binds to the nicotinic receptor, opens the Na channels, and eauses membrane depolarization, which results ‘ninsient fusciculations. Flacid paralysis wil follow in a few minutes, becanse snceinylcholine i resistant to acetylcholinestorase and will cause prolonged depolarization ofthe membrane. Phase H—Eventually the membrane will atleast partially repolarize. However, the receptor is now desensitized to acetylcholine, thus preventing the Formation of further action potentials In other words, suecinyleboline is now acting io manner similar to tubocurarine (Figure 7-1), What is the duration of Sito 6 minutes if given asa single dose action? ‘What substance metabolizes Flasina cholinesterase suecinyleholine? How is succinylcholine used As an adjuvant to general anesthesia clinically? ‘To facilitate rapid intubation 42 Section I Autonome Nervous System Phase Membrane depolarizes, esuling in an inal discharge which produces transient fascicultions falowed by Maced paralysis [Nicotinic receptor st euromuscular junction Na* Phase It Memiranerepoarizes but ecepor is Aesensized to elect of acetylcholine —— Succinst ‘hotine Figure 7-1. Mechanism of action of depolarizing neuromuscular agents (Redraven from Mycol PM), Gerener $8, Perper MM [Harvey RA, Champe PC, ede): ppnco Musee Rovews: Pharmacol, 2nd ed Phiadelphi,Lipincate-Raven Publishers, 1997, 9 53) Bronchoconstrietion eaused by histamine What are the adverse effect release Hypotension Arshythinias Apnea due to respiratory paralysis Malignant hyperthermia How is malignant hyper Dantrolene is used. It blocks the release thermia treated? ‘of Ca from the sarcoplasmic reticulin, hich subsequently reduees skeletal smusele contraction, De neuromuscular blocking agents block autonomie ‘ganglia as well? GANGLIONIC BLOCKERS ‘Chapter 7/ Cholinerrl Antagonists 43 In general, no, The skeletal muscle end plate and autonomic ganglia use different subtypes of nieatinie receptors. ‘Tubocurarine can, however, produce a small amount of ganglionie blockade Name four ganglionic Blockers. ‘What exactly do these drugs do? What is the mechanism of action? Describe the physiologic effects. 1. Nicotine 2, Hesamethoninen 3. Mecamylamine 4. Trimethaphan ionic inhibitors compete with acetycholine to ind with nicotinic receptors of both parasympathetic and sympathetic ganglia Ganglionic blockers can he divided into {90 groups 1. Drugs such as nicotine, which initially stimulate the ganglia and then block them because of a persistent ‘depolarization 2 Drugs such as hexamethonium, ‘mecamylamine, and trimethaphan, ‘which block ganglia without any prior stimulation ‘The physlalag effects of ganghole blockers an be preted you remember which dian ofthe lint conta fhe organ tn Heart~Tachycardia ret boas the pawjipatnes gem iver Chrminat onthe Beart, Asterioler and veine—Vasodiaton, Tncreared pecipbenl Hood Ceympathetic normaly ntinant) Bye Cyeoplegn yas, (pases ply iat) Gl gyatem “Tdoced mr ‘Bnrshed gute und parent secretions (gareympahetc normally beniant) 44 Section Autonomic Nervous System Urinary system—Urinary retention (parasympathetic normally dominant) Sweat glands-Reduced sweating (sympathetic normally dominant) What is the therapeutic use? Because they lack selectivity, the ‘ganglionic blockers are very rarely used clinically. In the past, these drugs were used in hypertensive emergencies ‘What are the adverse effects? ‘The toxicities of ganglionic blockers are identical to their physiologi effects, whieh have been deseribed above, Adrenergic Agonists What are adrenergic agonists? Drugs or endogenous catecholamines How can these substances be classified? Name the important o sel- ective direct-acting agonists. Identify the major fb select- ive direet-acting agonists. List the major « and i direet- acting agonists, Which of the direct-aeting, agonists are considered catecholamines? Name two indireet-acting, adrenergic agonists [Name two mixed (direct and indirect) agonists. that activate «and/or B receptors, These drugs are also knows as sympathomimeties. According to mechanism of ation (direct vs indizeet) as well as receptor site specifiy (ay, a By. Ba) Phenylephrine Methoxamine Clonidine Methyldopa Dobutami Isoproterenol Albuterol Metaproterenol ‘Terbutaline Epinepiaine Norepineplirine Dopamine Epinephrine, norepinephrine Isoproterenol, doparnine, and dobutan ‘Tyran and amphetamine Ephedrine and metaraminol DIRECT-ACTING « SELECTIVE RECEPTOR AGONISTS Where sare a and oy receptors located? ‘4, receptors are located on the effector ‘organ’s postsynaptie membrane. ‘4, receptors ate predominantly located ‘on the presynaptic membrane, 46 Section Autonomic Nervous Spree Postsynaptic a receptors are limited to the CNS and blood vestels, ulation leads to the release of lar caleam from the endo- What physiologic responses a, ‘occur when «receptors are intracell stimulated? plasmic reticulum via inositol triphosphate (IP) This leads to vascular constriction, decreased intestinal tone and motility, contraction of the bladder’s internal sphincter, ejaculation, ccntraction of the pregnant uterus, and mydriasis. (See Table 5-1 in Chapter 5— Introduction 9 Autonomic Nereous System Pharmacology.) When a, presynaptic membrane receptors are stimulated, intracellular cyclic adenosine monophosphate (cAMP) production is inhibited, a, receptors fietion primarily as part of a negative feedback loop. When norepinephrine is released from nerve terminals, some will circulate back to the presynaptic ‘membrane and bind tothe a receptor. ‘This wil subsequently inhibit further epinephrine release, Other actions mediated by ay receptors include increased vagal tone, platelet aggregation, snd suppressed insulin secretion. See Figure 8-1 Name the direct-acting 4, receptors—phenylephrine and agonists that are selective methoxamine for @-and a-adrenergic _ ag receptors—clonidine and methyldopa receptors. (discussed in Chapter 20—Antilyper- tensive Drugs) PHENYLEPHRINE (Neo-Synephrine) What are phenylephrine's Primarily vasoconstriction, The physiologic actions? suhsequent rise in blood pressure also leads to a reflex bradycarci, Deseribe this drug's thera- Asa nasal decongestant (primary use) peutic uses. ‘To treat hypotension For ocular examinations (mydriasis) ‘To terminate episodes of paroxysmal atrial tachycardia (PAT) Chaptor 8/ Adrenergic Agonists 47 12, Receptors, ‘Activation of eceptor decreases production Of cAMP, leading to an Inhibition of further release of norepinepiine from the neuron. ‘a, Receptors, ‘Activation of receptor ‘increases production of and inositol triphosphate, leading to ‘an increase in intracoluar calcium ions, Figure 8-1. Second messengers mediate she elects of w receptors. DAG ~ dicyebe- “oro P= inoskol phosphate. (Redraw from Mycek MJ, Gereer SB, Perper MM [Far- ‘vey RA, Champ PC, ed] Lippinct lrtrated Reviews: Prmacolgy, 2nd ed. Philadelphia, Uppincare Raven Publishers 1997, p 58.) 48. Section It Auronomic Nervaus System What are the adverse effects Rebound mucosal swelling andl associated with phenyl- hypertensive headache ‘ephrine administration? METHOXAMINE What receptors does As with phenylephrine, methoxamine is methoxamine work on? fainly specific for a, receptors Deseribe the therapeutie uses. ‘Trey ent of hypotension and PAT What are methoxamine’s The adverse eflects are similar to those of adverse effects? phenylephrine CLONIDINE Clonidine is also discussed in Chapter 20—Ansihypertensive Drugs. What receptors does cloni- stimulates ey receptors inthe dine work on? CNS, which reduces sympathetic nervous system outflow from the brain Deseribe its therapeutic uses. “Trealment of hypertension Withdrawal from benzodizepines and ‘opiates “Treatment of diarrhea in disbotie patients ‘who have autonomic neuropathies ‘What toxicities may patients Sedation ‘experience while using Dry mouth clonidine? Sewual dysfunction Orthostatic hypotension DIRECT-ACTING | SELECTIVE AGONISTS: Where are the B receptors receptors are primarily located on the located? postsynaptic membrane. By receptors une Found on both the pre-and postsynaptic rmomnbranes, What are the physiologic stimulation setivates adenylate responses once fi reeeptors cyclase, which apens caleium channels, imulated? Teading to cardine stimulation with both Inereased inotropic and chtonotropie effects. By stimulation also Teads to Increased lipolysis. By reeeptors work via adenylate eyclase stimulation as wel, DOBUTAMINE What is ie? What receptors docs dobutamine act on? What are the physiologic effects of dobutamine? What is dobutamine’s thera- pectic us What is the route of admin- istration? What are this drug's adverse effects? ISOPROTERENOL Which receptors mediate the effects of isoproterenol? ‘What are its physiologic actions? Chapter 8 Adrenergic Agonists 49 In this case, however, bronchial smooth is wells skeletal rnuscle ilatel The uterus, ciliary, und detrusor ruses are rebel muscle vasculature and glucagon release is increased. Both , and fi, receptors produce decreased intestinal tone and motility (just as the adrenengie receptors do), See aso Table 5-1 in Chapter 5—Intvoduetion to Auto nomi Nercous Systens Pharnuacoo A dopamine analogue Primarily, but it does have some aetion ‘on Bi receptors us Well Increased heart rate and contractility (B,) Siqoth muscle relasation ¢B.) ‘Treatment of unstable CHF and shook wv Armhythnias Headache Hypertension Palpitations Angina Nansea B, and B, receptors Inereases cardiovascular inotropic and chronotropic response (B,) Lowers peripheral sascular resistance B.) Relases smooth muscles (B,) Stimulation of heart rate in patients saffering from heart Block and bradycardia In the past, used for treatment of asthina ‘50 Section Autonomic Nervous System What is the route of ad Ww istration? What are the toxicities of | Armbythmias isoproterenol? Palpitations Tachyeardia Headache ALBUTEROL, METAPROTERENOL, AND TERBUTALINE What are the pharmacolagie Stimulation of smooth rmuscle dilatation actions of these fy direct Can stonilate receptors at higher acting agonists? doses What isthe route of admin. Albuterol und metaproterenol are usually istration? inhaled. Terbutaline ean be given orally or suberstanconsly: List the therapeutic uses. Treatment of hronchospasnasthina Treatment of chronic obstructive pulmonary disease ‘Treatment of bronchitis ‘Torbutaline and ritodrine can be sed to rolax the uterus during premature labor. What are the potential Arrhythmias adverse effects of the fy ‘Tachyeardin selective drugs? Headache Nausea and vomiting DIRECT-ACTING « AND & AGONISTS EPINEPHRINE Which receptors does Tt stimulates a, By, and Be receptors. epinephrine act upon? At low doses epinephrine stimulates & receptors and at high doses it stimulates receptors. What are the physiologie _Cardiovaseular—inereasod heart rate and responses to epinephrine? contractility: vasoconstriction of arterioles in the 3 ‘meous membranes Respiratory—bronchod scivation of By receptors Metabolic—ineressed gheogenolyss and release of ghicagon and «decreased ‘What are the therapeutic ‘What are epinephrine’s adverse effects? ‘To what does the term “epinephrine reversal” refer? NOREPINEPHRINE (Levophed) ‘What receptors does nor- epinephrine stimulate? What are its physiologic effects? What is its therapeutic use? What are norepinephrine’s adverse effects? Chapter 8 Adronergie Agonists 51 release of insulin results in hyperglycemia Given for bronchospasm secondary to cute asthma or anaphylactic shock Used in anaphylaxis and eardiae arrest to increase cardae electrical activity Used in conjunction with local anesthetics to prolong effects via vasoconstriction Used to achieve hemostasis, Cardiac arshythmias Hypertension Palpitations Dizziness, anxiety, headache Tremor Myocardial infarction due to increased ‘cardiac work Polmonary edema When epinephrine is administered alone, itil enuse an increase in systemic blood pressure hecanse ofits a activity. When sven in conjunction with an « blocker such as phenosybenzamine, epinephrine will cause a decrease in blood pressure because ofits activity. This effect is Inows as epinephrine reversal. 4, 3, and B, receptors. Norepinephrine has a stronger affinity for a receptors than {or B receptors Vasoconstriction Reflex bradyeardia Itis one of the last-line agents in the treatment of shock. ‘Tissue hypoxia secondary to potent ‘vasoconstriction Decreased perfision tothe kidneys ‘Tissue necrosis duc to extravasation during intravenous administration Arshytheias 52. Section l/ Auronomic Nervous Stern DOPAMINE Where is this agonist found? 11s synthesized in the C) i, and aren S, sympathetic rectal What receptors does 4, By, and BI also stinates its own dopamine act on? dopamine (D, and D,) receptors located inthe peripheral mesenteric and renal vascular beds, Dopamine receptors are stimulated a low dose, B receptors at moderate dose, and a receptors at higher doses. Dopamine does not eross the blood-brain barter. “Treatment of shock it raises blood pressute by stinnulating the B receptors of the heart Used in acute renal failure to inerease nal blood flow ‘Treatment of acute congestive heart failure v What are dopamine’s Decreased renal perfusion at higher adverse effects? doses Arshythinias Tachyeardia Hypertension ‘Tissue necrosis ean oceur f dopamine textnivasates during infusion, ECT-ACTING AGONISTS TYRAMINE ‘What is tyramine? ‘Tyramine isa by-product of tyrosine ‘metabolism; tyrosine i a precursor to dopamine, epinephrine, and norepin- ephrine, What is the mechanism of | “Tyraine is taken up by sympathetic action of tyramine? neurons, which eauses a release of catecholaanines Is there a therapeutic use No for tyramine? Chapter 8 Adrenerpl Agonists 53 What are tyramine’s adverse It con cause a hypertensive emergeney effects? ig patients who take MAO inhibiting drugs since MAO is responsible for the metabolism of tyramine, tis important to warn patients who are taking MAO inhibitors not to eat foods with high tyramine concentrations, such as red swine, beer, chocolate, and cheese AMPHETAMINE What are its pharmacologic It re action dopamine. Itcan enter the CNS, When is it appropriate to. Amphetamine is used to treat attention administer amphetamine? deficit hyperactivity disonler (ADHD) and narcolepsy. Ii also used for appetite suppression. ‘What are the adverse effects? Psychological and physical dependence Paychosis Confusion Insomnia Headache Restlessuess Palpitations Tachycardia Impotence MIXED (DIRECT AND INDIRECT) AGONISTS EPHEDRINE How does ephedrine work? It stinnuates th norepinephrine from nerve terminals. slo cts leuse of direct adrenergic agonist What are ephede Ephedrine is used in the treatment of therapeutic uses? urinary incontinence, bronchospasm, and hypotension What are the adverse effects? Archythmias Palpitations Tosornnia Hypertension 54 Section Il/ Autonome Nervous Sytem METARAMINOL, Deseribe this drug's actions. Metaraminol acts indirectly by releasing norepinephrine. It ean also directly stimulate cc receptors, What are its therapeutic ‘Treatment ofhypotension and uses? termination of PAT episodes ‘What are the adverse effects? Similarto those of norepinephrine bles Adrenergic Antagonists What are adrenergic ‘They are drugs that bind to adrenergic antagonists? receptors bit do not initiate the usual acellular rep 1e the wo major sub- 1. a blockers ns of this drug class. 2. B blockers wre another class of Yes—the indirect adrenergic antagonists adrenergic antagonists? @ BLOCKERS Name six @ blockers. 1. Prazosin (Minipress)— selective, reversible 2, Doxazosin (Cardura) —ay adrenergic lective, reversible erazosin (Hytrin) — are selective, reversible drenergie wie 4. Phenosybenzamine (Dibenzyline) — nonselective, irreversible Remember, phenoxybenzamine is the only ovadrenergic receptor ‘mentioned that is nonreversible board question) Yohimbino(Yocon)—ay-adrenergie sclectve, reversible 6. Phentolamine (Regitine)— nonselective, reversible PRAZOSIN, TERAZOSIN, AND DOXAZOSIN, What is their mechanism of ‘They competitively and selectively block Describe the physiologic Blockade of a-adrenergic reveptors on sequelae of e, blockade, ‘raseular smooth musele inhibits constriction of arterioles and veins. This results in decreased peripheral vascular resistance and a lower blood pressure ss 'S6 Section Il Autonomic Nervous System What are the elinical uses? Ave there adverse effects? PHENOXYBENZAMINE How does this drug work? Deseribe the physiologic actions of phenoxybenzamine, How is phenoxybenzamine administered? What is the duration of action? Describe the therapeutic use. Blockade of a-adrenergic receptors in bladder smooth musele results in relaxation and decreased resistance to urine flow ‘Treatment of hypertension Prevention of urinary retention in patients who have benign prostate hypertrophy Prazosin and structural analogues can Gastrointestinal hypermotlty Orthostaic hypertension, especially alter the inital dase Sexual dysfinction, dry mouth, and dizziness Phenosyhenzamine is mnie in that i works by noncompetitive blocking the 1 postsynaptic receptor and ay presynaptic receptors, It blocks peripheral yasoconstrietion Teinduees a reflex tachycardia Orally Because it hinds covalently to the receptor, this drug has a very long ‘duration of action (approximate hours) Treatment of patients with pheochromacytoma-induced Iypertension. Phenoxybenzamine is very effective because of ts long, duration of action, ‘Treatment of patients with henig prostatic hypertrophy. Phenoxy- benzamine reduces the size ofthe prostate ‘Treatment of patients with spinal cord Injuries who may suffer from hhyperreflexia, which results in high blood pressure. Phenosybenzamine ‘blunts this response. What are the toxi ciated with the use of phe- noxybenzamine? YOHIMBINE Deseribe the clinieal use, PHENTOLAMINE ‘What is it? Describe the mechanism of action. How is this drug used clinically? What is the route of administration? What are the adverse effects of phentolamine administration? B BLOCKERS Chapter 9/ Advenergic Antagonists ST ‘Treatment of patients with Raynaud's disease Orthostatic hypotension Reflex tachycardia—IF severe, it may induce anginal pain: therefore phenoxybenzamine is contraindica inpatients with coronary disease Inhibition of ejaculation due to lack of ‘smooth muscle contraction inthe vas deferens A selective ay-receptor antagonist It's sometimes used to treat impoteney. Via ditect penile injection. An imidazole derivative Reversibly blocks a, and ay receptors Beeause it has a half-life ofonly 4 hours, phentolamine I used for the short-term ‘control of pheochromoeytoms-indueed hypertension, TV or IM—poory absorbed orally COrthostatie hypotension rointestinal stimula lead to peptic uleers "Tachyeardia, myocardial infarct arrhythmias due to reflex sympathetic response How are B blockers sub- classified? All ofthe B blockers are competitive antagonists; however, they ean he subgronped according to three major properties: 1. Selectivity of receptor blockade 58 SecionIl/ Autonome Nervous Syste 2 Possession of intrinsic sympathomimete activity 3. Capacity to block a-advenergle receptors i SELECTIVE BLOCKERS Name four selective fh, 1. Atenolol (Tenormin) blockers. 2, Esmolol (Brevibloc) 3, Acehutolol (Seetral) 4, Metoprolol (Lapressor) In general, @-blockers starting with A or Maro cardiosclective Is their B, selectivity No. At high doses these drugs will block absolute?” By reeeptors “These drugs are sometimes called cardiosclective because they lack the unwanted bronchaconstritor and hypoglycemic effects of nonselective blockers, Atenolol—Hypertension, myocardial warrant the use of cardio, infarction selective B blockers? Esmolol—Becanse ofits very short duration of used when immediate B blockade is needed, such as for thyroid storm, eis ‘only aciinistered IV Acebutolol—Hlypertension Metoprolol—tiypertension, anginal pai ‘myocardial infarction NONSELECTIVE B-ADRENERGIC ANTAGONISTS What is the prototype non Proprnol selective fs blocker? Name the pharmacologic Decreased cardiac tpt snd blood actions of nonselective pressure blockers. Reduction of sinus rate and conduction throng the atin Peripheral vasoconstriction Bronchoconstrition Remember, th cardioseletive By Blackers lack the bronchoconsrcive and hypogyeemic effets of nonselective blockers Decreased eycogenolyis and gs secretion Inerened VLDI-and decreased EDL sigon Chapter 9 / Adrenergic Aeagonisss 59 How is propranolol absorbed? This drug is almost completely absorbed What is the site of propran- olols metabolism? In what clinical situations are nonselective B blockers indicated? Name two other nonselective B-adrenergic antagonists. What is the clinical use of these two drugs? What are the adverse effects of nonselective fs blockers? after oral administration, but only ‘approximately 25% reaches the systemic Circulation because of first-pass ‘metabolism, ‘The liver Hypertension Angina, tachycardia Ari ‘Thyroid storm Acute panie syndrome Migraine headaches ‘Timolol and nadolol—They have extremely long half-lives (20 hours). ‘Treatment of glaucoma —They deerease ‘the production of aqueous humor by the ciliary body: Bradycardia Bronehoconstriction—eain result in an asthmati attack May hide warning signs of hypoglycemia ‘such as tachyenedia; therefore, its critical to monitor diabeties who are receiving f-blockers, Fatigue Depression Sexval dysfunetion B BLOCKERS WITH INTRINSIC SYMPATHOMIMETIC ACTIVITY Name two drugs that are classified ass blockers but also have some B-agonistie properties, Why ure these drugs ‘considered to be partial agonists? What are these two drugs used for? Are there any advantages to using these agents? ‘Acebutolol and pindolol ‘They very milly stimulate both Band Byradrenergie receptors. However, their intrinsic effects ate not as strong as that of fall agonist, such as isoproterenol ‘Treatment of hypertension in patients prone to bradyeardia Acebutolo and pindolol produce bronchoconstriction only at extremely. 160 Section Autonomic Nervous System high doses. They do not induce badyeardia to the degree tht fll “antagonists do, and they cause very minimal disruption of lipid and carbohydrate metabolis 8 BLOCKERS WITH a BLOCKING CAPACITY Nonselective B-blockade along with ‘y-adrenengie selective blockade, which results in peripheral vasodilation rather than the vasoconstriction that aceurs with the other B blockers ‘Treatment of hypertension and atrial fibrillation What are the adverse effects? Orthostatic hypotension and diz Carvedilol (Coreg) What is it? AB blocker that also has a, blocking properties List the clinical uses. -eatment of hypertension Treatment of chronie CHF —Althongh it ‘may seem paradoxical to use blockers in the treatment of CHE singe they ean also worsen symptoms, they appear to Lenefit the patient by reducing sympathetic activity. They. nay also improve diastole dysfunetion by prolonging diastolic filling time. What are the mechanisms Reduction of sympathetie activity of action? Improvement of diastolic dysunetion by prolonging diastole fling time What are the contraindica- fi blockers are contraindicated in the tions to use? treatment of acute CHF. They are only used when the patient is hemodynam- ically stable a SELECTIVE BLOCKERS Butoxamine What is it? A selective Byradrenergic antagonist Does this drug have any No, not currently clinical use? NEW B-BLOCKING DRUGS Are there many other B blockers? How do these new drugs differ from the B blockers discussed here! How can physicians recognize these new drugs? What are the indications and adverse effects of the new B blockers? Cchopser 9 / Adronergle Antagonists 61 Yes, Now B blockers are produced yearly Primarily in their pharmacokinetics By the -olol ending in their names Generally they will be similar to those of other B blockers. INDIRECT ADRENERGIC ANTAGONISTS Why are guanethidine and reserpine considered indirect adrenergic antag- nists? (GUANETHIDINE (SMELIN) What is this drug’s mechan- ism of action? Does guanethidine have a clinical use? What are the adverse effects? RESERPINE What is it? What is reserpine’s mechan- ism of action? How is this drug used clinically? What are the adverse effects? ‘They do not drcetly block a- or B- adrenergic receptors. They do, hewever, block the release of norepinephrine from nerve endings—in elfect, they antagonize the effects ofthe sympathetic system. Itenters the peripheral adrenergic nerve by-a reuptake mechunistn for norepin- ephirine und binds to storage vesicles, the ‘action of which subsequently blocks the release of stored norepinephrine. Yes Onthostatic hypotension and sexual dysfinetion A Rawwofia alkaloid It blocks norepinephrine transport from cytoplasm into intracellular storage vesicles. Subsequently, the neuron isnot able to release any catecholamines. For treating hypertension (very rarely used) CNS depression and bradycardia dev 1 th aaa EdD! | ala de sea Satoh owt THIMOOATKA Simiauean A ToaRi Fine sihettans ppacrmiecka Koma fasjea0 eLrintny emailer iat of tate emits sly soir iclod ee te, owetorin ae — et a ve lbh ‘elaine, TL ae mall ee Section Ill Central Nervous System qvovial4 levinsD lil noftagd - niameye ; Name the major CNS neurotransmitters, What types of receptors are most commonly found in the CNS? What are the primary functions of a neurotrans- miter? What are EPSPs? Give five examples of excitatory neurotransmitters. What are IPSPs? Give two examples of inhib- ory neurotransmitters. Introduction to Central Nervous System Pharmacology Acetylcholine Newepinepliris Dopamine Serotonin ‘Gamma-aminobutyrieaeid (GABA) and lycine—newtral amino acids ‘Glutamatefaspartate—acidie amino acids Ton-gited revoptors (Na, K*, Ch, Gi) To bind a receptor and subsequently cither excite or inhibit the postsynaptic Excitatory posts Initiated when y transmitter activates Na or Ca channels L. Norepinephrine 2. Dopamine 3. Acetylcholine 4. Chutamate 5. Aspartate Inhibitory postsynaptic potentials initiated when an inhibitory neuro- trunsmitter opens chloride channels and the cell membrane hecomes hyper- polarized, IPSPs make it mone dificult for the neuron to become activated (Figure 10-1), L. Ghyeine 2 GABA “6 66 Section lf Central Nervous Sytem PsP Time Figure 10-1. Interacton of exciatory and inhibitory synapses. On the eta suprathresh- ‘ld seul is given co an excitatory pathway (E). On the righ this same stimulus is given shortly after stimulating an ihbitory pathway (hich prevents che excitatory potent from reaching theeshold,(Redeawn from Katzung BG: Base and nal Pharmacology, Teh ed, Stamford, CT, Appleton & Lange, 1998, p 345) In general, how do drugs Most drugs will affect production, affecting the CNS work? storage, release, or metabolism of a neurotransmitter. Other agents may allect the postsynaptic receptor. What are the major differ- ‘There are three major differences: ences between the autonomic 1. ‘The number of netrotransmitters is nervous system and the greater in the CNS, central nervous system? 2. The number of synapses is greater in the CNS, ‘The CNS, unlike the autonomic nervous system, has a large array of inhibitory neurons that serve to modulate action, Define ansiety. What are some of the physical symptoms seen with ansiety? What are the major classes of drugs used to treat anxiety? BENZODIAZEPINES Give some examples of benzodiazepines and their approximate duration of aetion. Anxiolytics, Hypnotics, and Sedatives An umpleasant emotional state consisting ‘of apprehension, tension, and feelings of danger, without a real or logical eause ‘Tachyeardia ‘Tachypnes Sweating ‘Trembling Weakness Benzodiazepines—the most frequently nised digs for ansiety Azaspirones—for example, buspirone Carbamates—for example, meprobarnate Barbiturates—rarely used today because ‘of severe side effects and a low therapeutic index. These drugs have generally been replaced by the Ddenzodiazepines Short-Aeting (2-8 hours): COxazepam (Serax) Clonazepam(Klonopin) idazolam (Versed) ‘Triazolam (Haleion) Intermediate-Acting (10-20 hours): ‘Temazepam (Restor) Lorazepam(Ativan) Alprazolam (Xanax) o {68 Section / Central Nervous Sytem Long: Acting(1-3 days) Chlordiazeposide(Librium) Diazepam (Valium) lorazepam (Dalmane) What is GABA? ABA (y-aminobutyric acid) is the major hibitory neurotransmitter of the CNS, How do benzodiazepines When benzodiazepines bind to speeifle work? receptors that are separate from bu adjacent to the GABA, receptor, they potentiate the binding of GABA to its ‘own receptor. The binding of GABA to its ‘own receptor results in increased chloride e, cell membrane yperpolurization, and decreased tation of action potentials, ber that benzodiazepines do not bind to GABA receptors—they bind adjacent to them (Figure LI=D) These drugs are used clinically at muscle indications for benzodiaz-_relasants and inthe treatment ofthe cepines? following: Anaiety disorders Panie disorders alprazolam i the drug of choice Stats epilepticus—diazepam isthe drug of choice Sleep disorders Tnsomnia—All benzodiazepines ean be sedating, but lorazepam and temazepam are the most commonly used Aleohol withdrawal—diszepam most ‘commonly used What is their route of PO, 1V, or IM administration? Where are benzodiazepines ‘They are metabolized inthe liver and metabolized? excreted in urine, Many of the Does dependence occur? —_Yes. Prolonged use can result in dependence. Abrupt discontinuation can result in withdrawal symptoms, including confusion, aniety, and agitation, Receptor Empty (No Agonists) Chori channel coseq). ST + \Uee Benzodiazepine receptor Empty receptor is inactive, and the eaupled chloride channe! is closed, Receptor Binding GABA Binding of GABA causes the chloride ion channel fo open, Receptor Binding GABA and Benzodiazepine Benzodiazepine Enity of Cr hyperpolarizes cell making it more dificlt te depolarize and therefore reduces neural exctabilily, Binding of GABA to its receptor is enfianced by benzodtazepine, resulting ina greater enty of chloride on, Figure 11-1. Schematic dagram of enzodazepine-GABA-chlarlde ln channe complex. GABA = +-aminoburyrc ace. (Redrawn from Mycek M), Gerener SB, Perper MM [Harvey RA, Champe PC, eds: ppc’ Mstrted Reviews Pharmacolgy, 2nd ed Pade, Lip incor: Raven Publishers, 1997, p91) 70 Section il Cantal Nervous System What type of symptoms may Deenssiness and eonfision—the most fa patient taking benzodiaz- common side effects epines experience? Ata Dizziness Respiratory depression and death, iF taken with other CNS depressants sich as ethanol BENZODIAZEPINE ANTAGONIST: FLUMAZENIL (ROMAZICON) What is flumazenil’s mecha-_Flamazenil (Romazicon) is « competitive nism of action? antagonist of benzodiazepines at the GABA, receptor. Deseribe the clinical use of Reversal of benzodiazepine sedation or this drug. overdose How long do the effects of Only 1 hour—Ropeat doses may be flumazenil last? necessary fora heavily sedated patient to remain alert AZASPIRONES: BUSPIRONE (BuSpar) How does buspirone work? Te acts as a partial agonist a serotonin (5 HT,,) receptors What are the indications Buspirone is used for generalized anxiety for this drug? however, unlike benzodiazepines, its celfocts may take 2 weeks to became apparent. What are the pharmacokin- ‘This drug is metabolized by the liver and tie properties of buspirone? excreted in the urine; its half-life is 2t0 LL hows How do the uetions of Buspirone lucks the muscle relaxant and buspirone differ from those anticonvulsant properties of the benzo. of the benzodiazepines? diazepines. What advantages does Mininnal sedation buspirone have over benzo- Low abuse potential diazepines? No overdose fatalities reported No witherawal symptoms What toxie effects are Headache, nausea, dizziness associated with buspirone? CChaptar II | Ansloytis, Hypnotes and Sedaoves 71 ‘CARBAMATES: MEPROBAMATE What is meprobamate’s [tis not well known mechanism of action? What is the elinieal use? tis now virtually obsolete, Inthe past it ‘was used primarily inthe treatment of ansety What are the adverse effects? Respiratory depression —mujor taxie effect Hypotension Shock Heart failure BARBITURATES Give four examples of 1. Phenobarbital (Lumninal) long. barbiturates. acting 2 Pentobarbital (Nembutal) —short acting 3. Amobarbital (Anmytal)—short-acting ‘Thiopontal (Pentothal)—ultrashort= How do these drugs work? Like benzadiavepines, barbiturates facilitate GABA setion on chloride entry into the cell, which results in membrane yperpolarization and a deerease in notron excitability, Barbiturates do net however, bind to benzadiepine receptors What are the therapentie Induction of anesthesis—thiopental ions for barbiturate Anticonvulsants—e.g., phenobarbital ‘administration? Treatment of ansiety Induction of hypnosis Why are benzodiazepines Benzodiazepines have a much hi favored over barbiturates therapentic index than do barbit for the treatment of ansiety? (Figure 1-2). By what routes ean barbitu- IV, PO, or IM rates he administered? What are the pharmaco: They are metabolized inthe liver and kinetic properties of barbi- excreted in the urine turates? 72 Secon Il Central Nervous System Barbiturates, scone ~» }ooma A a ? | Mosuitary depression / e Benzodiazepines 3 | anestesia a 2 | Hypnos z sbaation, ansotysis Increasing dose > Figure 11-2. Comparison of dose-response relationships of baodiaepines and barbie turates(Redrawn from Gall G, Hann CL, Hewson WH: The Pharmacdaey Companion ‘lore & Oriented Publishing Company, 1997, p 3, Fig 3.1) What determines the Redistribution tothe other tissues duration of action of thio- ental? Does barbiturate depend: ‘Yes—abrupt cessation can lead to severe withdrawal symptoms (tremor, estes ness, nausea, seizures, and cardi arrest For whom are barbiturates contraindicated? What are the adverse effects of these drugs? OTHER SEDATIVES For patients who have acute intermittent porphyria, because they increase porphy- sin synthesis Drowsiness and decreased motor control Induction of the P-450 system Addiction Respiratory depression and coma in high doses Allergic reactions, especially in patients ‘with asthma ‘ZOLPIDEM (Ambien) Deseribe this drug's clinical ‘Treatment of insomnia Chapter 11 Ansilyics, Hypnocis, andSedatves, 73 What are its adverse effects? Ataxia and confusion CHLORAL HYDRATE What are its linieal uses? Hypnosis Sedation (in children) Deseribe this drug's adverse Gastrointestinal distress effects. Unpleasant taste I What are antipsychotic drugs? What is their mechanism of action? Do antipsychotic agents differ in potene Do antipsychotics differ in cfficacy? How are antipsychoties ‘usually administered? Describe the absorption and metabolism of the tra- ditional antipsychoties. What is the onset of action? ” Antipsychotics Antipsychotis, also known as nero: leptcs, are drugs used primatily to treat psychotic states such as schizophrenia, delusional disorder, and other halhucinse tory states, Antipsychotics block varios receptors g cholinerje, ac receptors. However, their antipsychotic ‘actions are thovaght to be due to blocking, of dopamine receptors in the central nervous system, particularly the Dy receptors in the mesacortical and meso- mal systems of the brain, Yes—a drug's poteney parallels its allinity for Dz receptors. Haloperidol and thio- thivene are high-potency drags bee they have high affinity for the Dy recep. tors, whereas chlorpromazine and thio Fidavine ure low-potency drags becuse they have low affinity for Dy receptors No! The traditional considered to be equivale ychotics are all inefficacy these drugs are given orally, they ean be given intramuscularly i the patient fs noncompliant. ‘They are variably absorbed orally st they pass into the brain easily and have «large volume of distribution, Metabolism ‘occurs by the cytochrome B-450 system in the liver. Antipsychotics may not become effective for several weeks. However, sedation and othr side effects ean ooeur rapidly Chapter 12 Antipaychotes 75 Can these drugs cure ill: No! Antipsyehoties only reduce the nesses such as schizophrenia? symptoms ofthe illness they cannot cure the illness. ww are the antipsychotics Classification is based on structural classified? differences. The major classes include phenothiazines, butyrophenones, diben- ‘osazepines, thioxanthines, and ber soxavoles ‘TRADITIONAL ANTIPSYCHOTICS PHENOTHIAZINES What are some examples of Chlorpromazine (Thorazine}—prototype phenothiazines? Fluphenazine (Prolisin) Trifluoperazine (Stelazine) ‘Thioridazine (Mellaril) Porphenazine (Trlafon) What distinctive side effects Pigmentary retinopathy does thioridazine cause? May cause cardiac arrhythmias and eo ‘uetion block BUTYROPHENONES [Name two drugs in this lass. Haloperidol (Haldal) Droperido (Inapsine) Other than psychotic states, ‘Tourette's syndrome for what can haloperidol be Huntington's disease used? Pheneyeliine overdose—drug of choice What type of side effect is Extrapyramidal side effects (% common especially pronounced with board question) haloperidol? DIBENZOXAZEPINES Name u drug that belongs Loxapine (Lositane) to this class. THIOXANTHENES Name a drug that belongs Thiothixene (Navane) to this class. 6 Seccion Il Central Nervous System CLINICAL USES AND SIDE EFFECTS ‘What are the clinical appli Traditional neurolepties have seve cations of traditional ant therapeutic uses, hut the most impo psychotic agents? ‘Treatment of any agitated or psychotic state, such as bipolar disease or schizophrenia (These cespecilly effective forthe positive symptomis of schizophrenia, such as delusions, thought disorders, ard Iallucinations.) Antiometie therapy due to blockade of dopamine i the chemoreceptor trigger zone (Thioridazine, however cannot be used for this purpose.) ‘Treatment of Tourette's syndrome: haloperidal ‘Treatment of intractable hiewups— cchloxpron Antipruritie therapy-—promethzine Thecause of histamine blockade) What's an easy way to With a few exceptions all ofthe tr remember the side effects tional antipsychotics have similar of the traditional antipsy- toxicities —namely, sedation, extra choties? pyramidal effects, anticholin id ic elfect, adrenergic efleets (hypotension) Doall traditional antipsy- No, The severity of chotics produce the same varies among the classes of antipsychotic degree of each type of side drugs. For evample, high-potency drugs effect? such as haloperidol and fuphenazine provduce the greatest extrapyrisnidal effects, ancl lns-poteney deugs such as thioridazine and chlorpromazine proxlice the highest anticholinergi effects (Table averse fect 14), Describe the toxicities of CNS sedation seen markedly with the traditional antipsychotic phenothiazines agents. Endocrine alteration salactorrhien amenorrhea, and infertility, likely de to blockade of dopamine rl tho pituitary Anticholinergic effects dry mouth constipation, urinary retention, and blurry vision sae from, Table 12-1. Neuroleptic Drug Side-Fffect Profiles Side Effects Extrapyramidal Antchonlinensic we Adrenensi Potency Drug Sedation Effects Effects HIGH Haloperidal 1 4 1 1 Fluphenasine 1 4 2 1 2 3 2 1 2 3 2 1 Lomapine 2 2 2 2 Chlorpromazine 4 2 3 4 Low Tiordirine 4 1 4 4 *1= low. 4 high srnowpkednuy /2 sendeu u 78 Section Il Central Nervous Sytem Antiadrenergie effects —watch for light-headedness and orthostatic hypotension secondary to a- adrenergic blockade, Phenothia can cause failure to ejaculate Extrapyramidal side effects—akathisia (motor restlessness), parkinsonian syndrome (bradvkinie rigidity, tremor), acute dystonic reactions (slow, prolonged muscle spasins of tongue, neck, face), neuroleptic malignant syndrome, and tardive dyskinesia (common boar question) What is tardive dyskinesia? Tardive dyskinesia symptom that may ‘eeu ater prolonged therapy with neuroleptics (6 months to 1 year). Kes charaterized by rhythmical involuntary rmoverments ofthe tong, lips, oF jaw Pationts may also demonstrate puckering ofthe month, or even chewing rovenients Istardive dyskinesia ‘There is no known treatment for reversible? established cases of tardive dyskinesta, ‘The syndrome may remit partially or completely ifnenroleptie treatment is withdrawn, although in many eases it lrmeversible, Anticholinergie agents actually increase the severity of tardive Ayskinesia What is neuroleptic malig- Patients who receive neuroleptics for rant syndrome? long-term treatment may experience rigidity, altored mental status, cardiae arythiias, hypertension, and lfe- threatening hyperpyresia. ‘What is the therapy for This disorder is treated with dantrolene, a neuroleptic malignant skeletal musele relaxant (%. common syndrome? Dboard question). ATYPICAL ANTIPSYCHOTIC DRUGS. Name three examples of 1, Clompine (Clozaril) atypical antipsychotic drugs. 2. Risperidone (Risperdal) 3, Olanzapine (Zyprexa) Why are these drugs considered “atypical”? Describe the actions of clozapine. What is it used for? What are the side effects? Describe the actions of risperidone. Deseribe the actions of olanzapine. Chapter 12/Antipsychovcs 79 In addition to blocking dopamine receptors, atypical antipsychotics also produce significant blockade on serotonin HT) receptors. They also are mately associated with extrapyramidal side eects, ‘This agent is a dibensodiazepine derivative. I differs from traditional ‘antipsychotics in its potent blockade of serotonin (5-HT,) receptors along with the usual dopamine blockade, Clezapine has been effective in treating teases of schizophrenia that are refractory tocother neuroleptic drugs. Itis especially feflective in treating the negati symptoms of schizophrenia blunted emotion, withdraval, redueed ability to form relationships) Clozapine casses fewer extrapyramidal side effeets than traditional neuroleptics. Clozapine does cause seizures and a very Jangerous agranulocytosis in 1% to 2% of patients (2 common board question). ‘Weekly blood tests are required for patients receiving clozapine therapy Risperidone (Risperdal) a benzisoxazole drug that, ike clozapine, has a very igh affinity for 5-H, receptors. It also has ancidopaminergie (D,) activity. However, risperidone exhibits no anticholinergic effects and diminished extrapyramidal cffocts. Risperidone isa first-line agent for the treatment of schizophrenia since it 1s effective for both the postive and negative symptoms of the disease. The drug is ako known to prolong QT intervals and therefore should be used ‘with caution in patients who have abnormal QT stervals, Like risperidone and clozapine, lanza pine blocks both dopamine and serotonin receptors. Effective in the treatment of schizophrenia, it can produce anticholin ceric effects as wel as sedation and orthe- static hypotension, ANTIDEPRESSANTS Drugs Used to Treat Depression and Mania What depres What does the bioge amine theory of depression propose? List the m: antidepressants, ategories of Which of these agents are now considered first treatment for de TRICYCLICS An alfective symptom characterized by intense sadness, general loss of interest in the everyeay aspects of hfe, insomnia, changes in appetite, and low sell-estecrn, That depression is due toa deficiency of norepinephrine, serotonin, and dopamine in the synapses of the CNS. Tricyelies Serotonin-specifie reuptake inhibitors (SSRIs) Monamine oxidase inhibitors (MAOIs) Atypical antidepressants depressants ‘Tertiary Amine Trieyclies Anitriptyline—prototype Innipramine—prototy Dowepin Clomipramic Trimipramine Secondary Amine Trieyelies Amoxapine Maprotline Protriptyline Desipramine Nortriptyline (Chapter 3 / Drvgs Used to Treat Depression and Mana 81 What are the physiologic ‘The secondary amine tricycles in general differences hetween tertiary arc les likely to cause sedation, hypo- amine and secondary amine tension, and anticholinergic effects twieyelies? However, they are more likely to induce psychosis What isthe mechanism of ‘These drags are thought to increase levels action of all the trieyelies? of norepinephrine and serotonin in th upc let by blocks reuptake, They also lack histamine, cho. linergie, and a-adrenergic receptors, which accounts fora linge proportion of their side effects, Trieycles are wl thought to ¢ ogulation of yonoamine receptors this may account ea dow for some of their therapeutic benefit No. These drugs are not CNS stimulants, ‘Mood disorders (depression prin Panic disorder Generalized ansiety disorder Posttraumatic stress disorder Obsessive-compulsive disorder {clomipramine} Pain disorders :muresis in childeen (imiprasmine) How are tricyclics admin- They are well absorbed only, and istered? penetrate into the CNS easily How are these drugs ‘They undergo significant first-pass metabolized? metabolism inthe liver: they ate con jiugated with glucuronie acid and excreted thromgh the kidneys. Which of the trieyelies are Allare exqullyefficicions most efficacious? ‘When should a physician Although the uptake mechanism is expect to see a change in ibited almost immediately, ant the patient's mood? depressant clinical effects may require 2 to weeks to became apparent. This suggests that their mechanism of action is rot completely understood, 82 Section il / Central Nervous Sytem What are the signs and Anticholinerge side effects blurred smptoms of trieyelic vision, hot dry skin, constipation, toxicity? confusion, urinary retention Autonomic effects —orthostatie hypotension ECG changes—widening of the QRS ccomples, arrhythmias Weight gain Sedation due to histamine blockade Possible lowering of seizure thresholds Can tricyclics and MAOIs No! There isa chance that this combi: bbe given together for added nation can lead ta severe convulsions and benefit? coma, ‘SEROTONIN-SPECIFIC REUPTAKE INHIBITORS (SSRIS) Give some examples of SSRIs. Fluoxetine (Prozac) —prototype Sertraline (Zoloft) Paroxetine (Paxil) Fhuvoxamine (Lavox) What is their mechanism of Inhibition of serotonin reuptake without action? significant effects on norepinephrine and dopamine ‘When would these drugs be Depression is the primary reason for indicated? preserihing these drugs, Fluoxetine is also sed to treat obsessive-compulsive disorder, How are SSRIs administered? Orally How are they metabolized? By the eytochrome P-450 system, Flavoxatnine isa potent P-450 inhibitor Deseribe the side-effect In general SSRs have fewer side effects profile of SSRIs. (anticholinergic, antiistaminie, ee adrenergic blockade) than do other classes of antideprestants What adverse symptoms Nausea may # patient taking SSRIs Diarthea i Nenvousnes Insomnia Dizziness Impotence Deeroased libido Chapter 12 / Drugs Used to Treat Depression and Mania 83 ‘When are SSRIs contra- SSRIs are contraindicated in combination indicated? therapy with monoamine oxidase {inhibitors (MAOIs) because this combi. nation may result in the “serotonin syndrome,” characterized by hyper- thermia, muscle rigidity, myoclonus, and rapid changes in mental status. MONOAMINE OXIDASE INHIBITORS (MAOIs) ‘What is monoamine oxidase? MAO {a mitochondrial enzyme that is Where are the highest con- centrations of this enzyme? Describe the mechanism of Two types of MAO exist: MAO-A and, action of the MAOIs. MAO-B. Within the neurons, MAO-A is responsible for the inactivation of any serotonin or norepinephrine that may leak out of presynaptic storage vesicles. When MAO-A is blocked, those neurotransmitters accumulate and are released into the synapse. MAO-B is responsible forthe metabolism ‘of dopamine and works ina similar ‘manner. In general, the MAOIs are nonspecific inhibitors of MAO, except for sclegiline, which Isa specific inhibitor of MAO-B, What are the MAOIs ‘Treatment of atypical depression (with indicated for? phobia or psychotic features). Other classes of antidepressants are more frequently used today because they have fewer toxic effects Deseribe the pharmacologic They are well absorbed orally properties of MAOIs, ‘They are metabolized by acetylation in the liver (halF-life, 2-3 hours). ‘They require 2to 4 woeks of treatment to reach a steady-state plasmna level What are the adverse effects Hypertensive crisis (headache, hyper of MAOIs? tension, arshythmias, and possibly stroke) ifthe patient does not avoid {84 Secon Il / Cantal Nervous Sytem cating foods high (cheeses, chicken liver, heer, and red wine). At increased levels, tyramine will release catecholamines from, storage vesicles and therefore will act asa pressor agent. Hypertensive criss ‘em also oceur when MAOIs are ulministered with meperidine, Onthiostati hypotension Day mouth, blurred vision ‘Weight gain ATYPICAL ANTIDEPRESSANTS. c two examples of atyp- 1, Trazadone (Desyre) ical antidepressant agents. 2. Bupropion (Wellbutrin) Why are these drugs con- ‘Their mechanism of action is ess well Alef than that ofthe tational ant= depressants An antidepressant similar in structure t0 the benzodiazepine alpeeolum (Xana), ‘but more specific for inhibition of sero. toni reuptake ‘To treat depression. It is particularly effective for improving sleep, In the liver (excreted by the kidneys) Deseribe the ndverse effects. Sedation Orthostatic hypotension Nausea Headache and dizziness Agitation Rare anticholinergic side effects Bupropion Describe this drug's mecha: It isnot clearly known, nism of action, What are the indications for Depression bupropion? (Chapter 13 /Druge Used to Troat Depression and Mania 85 What toxicities are asso- Headach ciated with bupropion? Nausea, Tachycardia Reatlessness What is the advantage for There are no side effects related to sexual using bupropion? dysfumetion, such as those that eceur with SSRIs. ANTIMANIC AGENTS: Elevated mood with grandiose ideas, cexpansivenest, pressured speech, light of ideas, decreased sleep, and inereased activity, In what conditions is mania _Alfective disorders such as bipolar seen? disorder Give some examples of Lithium—drag of choice ‘antimanie agents, Anticonvulsants (valprote acid and carbamazepine) —See Chapter 14— Anticonculsants for further discussion ‘of anticonvulsants uTHIUM What is it? A light alkali metal available in carbonate, slow-release, and controlled-release forms “The mechanism is unclear, iis thought to block the enzyme inositol-I-phosphatase, which affects neurotransmitter What is lithium’s major Itis primarily ued in bipolar disorder— idication? Doth manic and depressive episodes respond. Deseribe some other uses. This drugs also used as an adjuvant with antidepressants to treat major depression, sand with antipsyehoties to treat sehizo- phen It is well absorhod orally and excreted in the urine. What are the pharn kinetic properties of lit When is lithium contra Preynaney—it is teratogenie indicated? What adverse effects should physicians wateh for when administering lithium? What substances affect lithium plasma levels? How is lithium toxicity treated? Section Ill / Central Nervous System Acute intoxication—severe tremor, x seizures, confusion, and coma Nephrogenie diabetes insipidus ‘Weight gain, vomiting, abdominal cramps, and diarshea Disturbances in thyroid function (Monitor TSH for lithium-induced hypothyroidism.) Depression of T-wave on ECG Leitkoeytosis Excessive intake of sodinm lowers lithium levels ‘Thiazide diuretics increase plasma levels of the drug, Overdose are eleared by dureties. As previously mentioned, do not use ‘asides because they increase plasma levels ofthe drug. Use sox bicarbonate, or dialysis if necessary What is « seizure? 1 five major eauses of List the different types of What is « partial focal) seizure? What are the characteristics of a simple partial seizure? ‘What happens ins complex partial seizure? Where de complex partial seizures originate? Anticonvulsants An abnormal, synchronized, electrical \dopolarization of neurons in the central nervous syste Idiopathie CNS infections Fever Metabolic defects Cerebral tear, Partial Generalized tonic-clonic Status epileptions Absence Febrile Myoclonic A seizure in which abnormal discharges ‘cen fom a focal area within the Drain, ‘There ure two types of partial seizures: simple and complex. [A simple partial seizure involves focal neurologie symptom that can be sensory (or example, auditory oF visial hallucinations), motor, or psychomotor. Consciousness i always retained. The intial focus of abnormal discharge spreads so that the patient will lose ‘consciousness and have postital (post- scizire) confusion. Symptoms ean inclade ‘coordinated motor aetiviy hallucinations 1 olfactory ‘The majority originate in the temporal lobe, 7 {88 Section il / Cone Nervous Systm What part of the brain is involved in a generalized -clonie (grand mal) Name and deseribe the typical phases of a grand. mal seizure. Can a partial seizure develop into a grand mal ‘What is status epilepticus? What are the charncteristies of absence (petit mal) What are the characteristics of febrile seizures? What are the characteris of myoclonic seizures? Define epilepsy. What percentage of the population is affected by epilepsy? “The entire cerebral cortex Tonic phase—loss of consciousness, rigidity, loss of bowel and bladder control Clonie phase—jerking movements of the entire body Yos—this is known as partial seizure with secondary generalization Continuons seizes not separated by any periods af regiined consciousness, This condition is medical emergency “They usually eur in children 2 to 12 years of age ‘They are chaructorized by a very brief (10 seconds) loss of consciousness. The child will stop whatever he is doin and stare or have some facial toitching. Follascing the uttack, the child becon immediately alert and is seldom even aware that it has oveurted, ‘They occur in children They usually last less than 10 minutes, The child has.a fever, but there is no ‘apparent infection or other defined cause forthe seizure. "They ure sudden, short episodes of either local or generalized muscle ‘contractions ‘They ean occur at any age ‘They are associated with «variety of rare hereditary neradegenerative disorders, pilepsy isa group of chronic syndromes characterized by recurrent seizures \with periods of conscionsness. About 1% Chapter 14 /Anciconvuiants 69 ‘ypee ot Elepay Anticonvulsant Agents a a] = ENE GD tebe ee) [Preetwois) [mms awe TD EE ese se) rine NEE MEN cee) [Ram [Fini] EEE ee] a —a oe a es ee) (Uses reese Figure 14-1, Therapeutic indication for aniconvusaneagens. (Adapted and redrawn from Myce), Germer $8, Perper Mt [Harvey RA, Champe PC. ed): Lipinat hs trated Revews Pharmacsory, 2nd ed Philadephia, Uppincore-Raven Publishers, 1997, p 145.) What are the pharmacologic Thenytoin treatment options for Carbamazepine seizures? Phenobarbital Prienideme Valproie acid Ethosusimide Benzodiazepines tabapentin Lamotrigine See Figure 14-1 PHENYTOIN (DILANTIN) What are the therapeutic _Phentoin is eflectve in treating tonic- uses of phenytoin? clone seiaures and partial siz nt absence seizures Ibis also used in the treatment of status epilepticus alter the administration of diaoepam, bout “Whats the mechanism of | Phenstoin binds to and prolongs the action? wctivated state of Na* channels. Describe the absorption Oral absorption is slow. Phenytoin and metabolism of this drug. undergoes hylroxylation by the hepatic microsomal enzyme system. At high doses 90 Secon Il Central Nervous System when the hydroxylation systern becomes saturated you need to watch for toxicity What are the toxic effects? Gingival hyperplasia loblastic anemia secondary to interference with folate metabolism Hirsutis Diminished deep tendon rellexes in extremities ENS depression docrine disturbances—diabetes insipidus, hyperglycemia, glycosuria, osteomalacia Name three drugs whieh L. Chloramphenicol increase plasma concen- 2 Toninzid tration of phenytoin, 3. Cimetidine Name one drug which is Carbamazepine well known to decrease plasma concentrations of phenytoin. Is phenytoin teratogenic? Yes. It produces fetal hydantoin syndrome, which is characterized by prenatal growth deficiency and mental deficiencies, There is also an increased incidence of congenital malformations suc as cleft palate and heart malformations CARBAMAZEPINE (TEGRETOL) What is the therapeutic use Its the dmg of choice for treating partial of carbamazepine? and tonic-clonic seizares, Itis also the nugof choice for treating trigeminal neuralgia, What is its mechanism of It profengs the inactivated state of Na* action? chanmels, What are the absorption Carbamazepine is absorbed sly wh and metabolism of this drug? given orlly and is metabolized by the P- 450 system, Which drugs inhibit the Enthromiyein ism oF carbamar. —— Isomiarid metabo pine? Propoxyphene What are carbamazepine’s adverse effects? Chapeer 14 Anticonvulsants 91 Verapamil Cimetidine ‘Acute intoxication can lead to respiratory depression, stupor, or coma, Severe liver texiity—Patients need frequent Iver fmetion tests while recelving this drug, Aplastic anemia Ageanulocytosis Patients frequently complain of. drowsiness, ataxia, nystagmus, and vomiting HENOBARBITAL (LUMINAL) What is the classification of this drug? State its mechanism of action. What are phenobarbital’s therapeutic uses? What are the absorption ‘and metabolism of pheno- barbital? State phenobarbital’s adverse effects. PRIMIDONE (MYSOLINE) Teis a barbiturate Potentiation of synaptic inhibition through an action on the GABA receptor 1, tis the drug of choice for trating fobrile seizures it is also used to treat grand mal seizures in children, 2. It's good for treating partial seizmees and tonie-clonie seizures; however, its sedative effects have reduced is se as a primary agent, ‘The drug is well absorbed only, 75% of itis metabolized in the liver. Ibis a potent inducer of the eytochrome P-450 system, ‘The metabolic by-products are excreted in the urine Sedation Nystagmus Psychotic reactions Hypersensitivity react Johnson syndrome What drug is primidone structurally related to? Its related to phenobarbital and it works the same way as phenobarbital. 92 Section ll / Cenral Nervous Systm When is this drug sed? How is this drug metabo- lized? ‘What are its adverse effects? Primidone isan alternative choice for adults who have partial seizures (both, simple and complex) and generalized tonic-clonic seizures, Primidone is converted to phenylethyl ‘malonamide (PEMA) and to phenobar- Dita in the iver ‘This drug's toxi effects are very similar to those of phenobarbital: Sedation Ataxia Nauses Vomiting Drowsiness VALPROIC ACID (DEPAKENE) ‘What are the indications for use of this drug? How does this drug work? ‘What isthe route of admin- istration? How is it metabolized? What side effects should you wateh for when administer ing valproic acid? Should pregnant mothers be given valproic acid? Valproic acid is the most effective agent for treating myoclonic seizures, It also used in the treatment of ubsence sekzures, It prolongs the inactivated state of Na channels. It may aso increase GABA con ‘centrations inthe brain, ‘Valproie acd is well absorhed orally Once absorbed, approximately 90% of the dlrugis bound to plasma proteins The drug is extensively metabolized the liver by the eytochrome P-450 system, However, it does not induce the enzymes of this system. Approximately 3% of the dlnug is excreted imehanged Hepatotoxicity This drug fulminant hepatitis, which ean be fatal Nansea and vomiting Sedation, Tremor No! The incidence of neural tube defects is vory high f this drug is taken during the frst trimester of pregnancy CChapeer 4 /Antconvatianes 93 ETHOSUXIMIDE (ZARONTIN) ‘What is this drug used for? Ethosuximide is the drug of choice for ‘eating absence se What is its mechanism of ——Ethosusimido inhibits Ca? influ through action? ‘T-type channels inthe thalamic neurons What are the absorp! [tis well absorbed orally. The majorty of and metabolism of this drug? the drugs metabolized by the eytom ‘chrome P-450 system in the liver. It does not inhuce P-450 enzyme synthesis State the toxie effects Dizziness Agitation Gl distress Confusion Blood dyscrasis such as leukopenia astic anemia, andl thromboeytepenia may oceur in extremely sensitive pationts. ‘Skin reactions such as Stevens-Johnson. syndrome have been reported. BENZODIAZEPINES. See Chapter LI—Ansiolytics, Hypnoties, andl Sedatives for further detalls about benzodiazepines, Give three examples of 1. Diazepam (Vin benzodiazepines that are 2. Clonazepam(Klonopin) ‘used for anti-epileptic 3. Clorazepate (Transene) purposes What is the therapeutic use Intravenous diazepam is the drug of of these drugs? choice for initiating treatment of status epilepticus, Clonazepam can he used for teating imyoelonie seizures in children Slonizepate may be used for partial seiztes in combination with other drugs. State the adverse effects of ‘The benzodiazepines have relatively benzodiazepines. minor side effects, but you should watch for the following: ‘94° Section Ill/ Central Nervous System Drowsiness Respiratory depression Cardiac depression GABAPENTIN (NEURONTIN) State the therapeutic use Gulpentin is used to treat partial of this drug. seizures with and without secondary generalization. This drug is used in adults in combination with other antiseizure drugs. What is the mechanism of Gabapentin has been found to pr action? release of GABA ‘What is the metabolism of Its excreted unchanged in the urine, this drug? State the toxie effects. Ataxia Somnolence Fatigue LAMOTRIGINE (LAMICTAL) ‘What is the therapeutic use It is used to treat partial seizures in adults of lamotrigine? in combination with other druge. ‘What is its mechanism of Lamotrigine blocks sustained repetitive action? fring by blocking voltage-dependent Nat channels Name the site of metabolism, ‘This drug is metabolized in the liver: What are the toxie effets Dizziness of lamotrigine? Blurred vision Rash ARKINSON’S DISEASE Drugs Used to Treat Parkinson s Disease and Other Movement Disorders What is Parkinson's disease? What is the pathophysiology of this disease? What are the pharmacologie treatment options? Can these drugs cure? What is the treatment strategy? [A movement disorder that has the following four eardinal characteristics: 1. Resting tremors Muscle rigidity 3, Bradykinesia 4. Abnormal gait and posture ‘The disorder is thought to ceeur because ‘of alos of dopamine in the nigrostriatal pathway (Figure 15-1). The loss of dopamine disrupts the delieate balance Ihetween the cholinergic and deparnin fergie systems within the striatum and Daal ganglia, c Bromocriptine Penyolide Amantadtine Selegiline Antimuscarinie agents No! Pharmacologic treatments can only offer temporary relief; they neither “The ultimate goal is to reestablish the balance between dopamine and acetylcholine in the brain. Ths ean he 9s 96 Secon i / Central Nervous Sytem Sites of inhibition (-) oF ‘excitation () inthe striatum ‘and substantia nigra @ GABA aI ‘| honed Substantia nigra Dopamine neurons degenerate in Parkinson's disease because of ‘widespread destructon of substantia. ‘gta. This resus in 1ACh roloase Inthe neostratum and increased GABA input into the substantia nigra. Figure 15-1. feces of dopaminergic neuron losin Parkinson + dsease, DA = dopamine: GABA ~ >yarinobusyre ace: ACh ~ acazycholine.(Redrawn from Mycek M, Geren SB, Perper MM (Harvey RA, Champe PC, ed] Uppinct's Mastrted Reviews Pharmacology, 2nd ‘Philadelphia, Lippincor-Raven Publishers, 1997, p 84) plished by either (1) increasing dopamine in the nigrostriatal system, oF (2) reducing the cholinergic output ofthe striatum, LEVODOPA (Larodopa) What is it? “This metabolic preenrsor of dopamine isa first-line drug in the therapy for Park- inson’s disease. What is the advantage of Dopamine itself does not eros the blood brain barrier. However, levodopa is transported into the beuin and subse {quently converted to dopamine in the basal ging, Chapter 15 / Prkinon + Disease and Other Movement Disorders 97 What are the disadvantages of using this drug alone? What are the pharmaco- kinetics of levodopa? the on-off phenom- ‘enon? What drugs should NOT be given with levodopa? What are the adverse effets of levodopa? ‘CARBIDOPA (Lodosyn) ‘What isi? Why use it? Large doses of levodopa are required iit Is used alone because this drug is deear- hhoxylated inthe periphery to dopamine Levodlopa i typically used in combination with another drug such as carbidopa This drugis absorbed wel from the GL traet, However, if ingested with high- protein meals, the transport of levodopa tacrass the blood-brain barrier is impaired because of competition from neutral no acids Beenuse levodopa has an extromely short Fale, plasma levels may drop. suddenly. This may cause sudden immo bility, tremors, and eramps. The develop- ment of rapidl changes in clinical response to the drug is known as the on-off phenomenon. Nonselective monoamine oxidase inhibitors—This combination will result in exeess dopamine in the pe- riphery which could lead to a life- threatening hypertensive eriss. Pyridaxine—This drug diminishes the cffoctivenese of levodopa because it increases peripheral brealdown af the drug Antipsyehotiesthese drugs block dopamine receptors Nausea, vomiting, arrhythmias, and postural hypotension—due to levo dopa being comverted to dopamine in the periphery Dyskinesi, hallucinations, restlessness, ‘and confusion —due to ‘overstimulation of central dopa receptors A dopamine decarboxylase inhibitor that tloes not exoss the blood-brain barter ‘When administered with levodopa, cearbidopa reduces the metabolism of dopamine in the periphery and therefore 98 Secon Il / Central Nervous Sytem eases the availability of dopamine to the CNS. The addition of earbidopa reduces by fourfold the amount of leva dopa needed. What is the efficacy of “Two thirds of persons who follow thi levodopa/earbidopa regimen show some remission of symp- (Sinemet) treatment? toms (especially bradykinesia) inthe fist 2 years of treatment. Treatment efficacy declines as the disease progresses. This ‘occurs because levodopa requires some healthy dopaminergic neurons to be cflcctve BROMOCRIPTINE (Parlodel) Bromocriptine isan ergotaine derivative that acts as a dopamine receptor agonist at D, receptors. Tes a second-line drug after levodopa, Bromocriptine is used in conjunetion with levodopa. It may allow reduction in the levodopa maintenance dosage and therefore may reduce the oovurrence of side efleets associated with long-term levodopa use. Bromocriptine has very litte effect on park shen used alone; however, when used in inction with levodopa, it helps relieve akinesia, rigidity, and tremor. ‘used to treat itis the drug of choice to treat cases of hyperproluctin- ‘emia (% common board question) What are the adverse effects Hallucination, delirium, nausea, and of this drug? vomiting Cardiac arrhythmia, postural hypotension ythromelalgia PERGOLIDE (Permax) How does pergolide work? It works ina manner similar to bromocriptine; however, itis a dopamine agonist at hoth D, and D, receptors [Chapter 15 / Parkinson ¢ Disease and Other Moverent Disorders 99) What role does it have in Its usually used in combination with the treatment of Parkinson's levodopa disease? What are the adverse effects? Confusion Hallucinations Orthostatic hypotension Urinary tract infections SELEGILINE (Eldepry!) What is its mechanism of —_Selegiline selectively inhibits monamine action? ‘oxidase B (MAO-B), which metabolizes dopamine. Monoamine oxidase A (MAO-A), hovrever, isnot affected unless extremely high doses of selegiline are sven, ‘What is ts therapeutic use? Because it decreases the metabolism of dopamine in the periphery, slegiline increases dopamine levels in the brain. ‘The effects of levodopa are enhanced when used in conjunction with selegline: however, there is a threat of hypertensive crisis when this drug is administered in high dosages AMANTADINE (Symmetrel) ‘What is this drug's classif- Amantadine is an antiviral agent used to cation? treat influenza What is its mechanism of ‘The exact mechanism is unknown. ction? Amantadine appears to either enhance the release of dopamine from surviving nigral neurons or inhibit the reuptake of dopamine at synapses. ‘What is its therapeutic use? Amantadine may improve bradykinesia, tremor, and rigidity when used along with levodopa. It is usualy effective for only a few weeks but seems to be more effective than anticholinergic agents, ‘What are the toxie effects __Restlessness, agitation, confusion of this deug? Orthostatie hypotension Peripheral edema LLivedo reticularis (skin rash) 100 Section It Central Nervous System ANTICHOLINERGIC AGENTS Name three examples of The three most commonly used this class of drugs. anticholinergis are: 1. Benatropine (Cogentin) 2. Biperiden (Akineton) 3, Trihexyphenkdyl (Artane) Why use them? “These drugs help reduce the cholinergic ‘output of the striatum (see Figure 15-1), ‘Again, this fs another method to restore Dalince between dopamine and acetylcholine with the nigrostsatal system, What is their therapeutic These drugs ane mich less effleacions efficacy? than levodopa, They are commonly used aadjuvantly in parkinsonian therapy. They primarily help reduce tremor, rigidity and akinesia; secondary symptoms such as drooling are also reduced. What are their adverse “The following are caused by decreased effects? parasympathetic response Sedation Urinary retention Dry mouth Constipation Mental confusion ADDITIONAL MOVEMENT DISORDERS What is drug-induced Parkinsonian symptoms ean he eansed by parkinsonism? potent antipsychotic agents such as halo- pperidol because they black dopamine receptors What is the treatment for ‘There are three treatment options: ‘drug-induced parkinsonism? — 1. Lower the drug levels. 2. Change the drig to. less potent one, 3. Use an anticholinergic agent. Define Huntington's disease. A genetic disorder due toa single defect ‘on chromosome 4 What are the symptoms? Individuals who have this disease display dementia and/or chores, (Chapter 15 {Parkinsons Disease and Other Movement Disorders 101 ‘What is the pathophysiology? What is the treatment? What is Tourette's syndrome? ‘What is the treatment? Define Wilson's disease. What is the treatment for Wilson's disease? ‘This disease Is thought to oceur because of excessive dopaminergic activity and diminished y-aminobutyric acid (CABA) functions in the basal ganglia (caudate ‘and putamen) Dopamine blockers such as huloperidel (Haldol) or tetrabenazine are used to treat this disorder. A disease characterized by abnormal tes and facial movements Clonidine (Catapres), Haloperidel as also been used. Wilson's disease ia genetic dsorder of copper metabolism. Excess copper is dlepenited in the liver, brain, and other A copper chelating agent known as penicillamine (Depen) 1b What are the two major classes of anesthetic agents? Deseribe the routes of administration and primary actions of each of these two classes. GENERAL ANESTHETICS Anesthetics General and local General anesthetics are given either as inhaled or intravenous agents: they primarily have CNS effects. Local agents are injected at the operative site to block nerve conduction, What are the stages of ‘general anesthesia? How do the pharmaco- kineties of an these stages? What is induction of anesthesia? What is induction dependent upon? 102 ic ar aps Stage -AnalgestaRoduced sensation 1 pig he pata remoint eons and convertional, Stage H-Eeltement—Delirum and combative behavior ens, theres an esoaiatn bod jesse kad searanay tn Stage TH-Surgical anesthesia The Tents ecncions ad eg Fespration returns thee mule qalbstion cl docreaed soxchor response opin stim stage 1V-Medullary paralysis— Tesiatry drive decreases and sscomotr output diminishes; death equa ea With slow-onset agents (for example, ther), all four stages are discernible. Faster-acting agents allow for quicker progression through the stages, ‘The time from administration ofa general anesthetic to the achievement of surgical anesthesia How fast the anesthetic reaches the CNS. How are the complications of anesthetic induction avoided? What is recovery? What does recovery depend upon? For inhaled anesthetics, shat five factors influence the rate of induction? Deseribe how solubility affects the rate of induction. How does pulmonary vent- ilation affect the rate of induction? Describe how partial pressure of the inhaled agent affects the rate of induction. Chapter 16/Anesthenes 103 An ultra-ist-actng, short-lived agent (such as propofol) is given IV so that the patient will rapidly progress through the First and second stages of anesthesia Simply, the reverse of induction How quickly the anesthotic is removed from the: 1 2 y 3, Partial pressure of the inhaled agent 4, Alveolar blood flow 5, Arteriovenous concentration gradient ‘The blood-gas partition coelficient is an index of solubility alow coefficient implies relative inslubilty. An agent with a low solubility requires fewer molecules toraise the partial pressure of the agent inthe bloods thus, the equilibrium between alveolar partial pressure and arterial partial pressure is achieved rapidly, which leads to faster induction. Recovery is likewise hastened when the anesthetic agent is discontinued. ‘The rate and depth of ventilation the ‘minute ventilation) affects the rate of yerease in partial pressure ofthe ‘anesthetic inthe blood. An inerease in the minute ventilation results in an increased amount of agent, This effect is most important for agents with low solubility because they require greater ‘amount of agent to achieve equiltrin, An increased concentration in the inhaled air misture leads to greater concentration atthe alveoli and thns increases the arterial partial pressure of the agent. In clinical practice, a greater concentration {s given initially to speed induction, and then its reduced to 4 maintenance level 104 Section il / Central Nervous Sytem How does alveolar blood flow affect the rate of indueti Deseribe how the arterio- ‘venous concentration affects the rate of What factors influence tissue uptake of an anesthetic? What is the molecular mech- anism of action for general ‘anesthetios? INHALED AGENTS Name six inhaled agents. How is the poteney of inhaled anestheties defined Inereased flow allows for more rapid uptake of the agent and quicker effect on the CNS, ‘This is dependent on the uptake ofthe at by issue. A high rato and extent of tissue uptake will decrease the venons ‘concentration ofthe anesthetic. Asa result, it will take a longer time for the anesthetic concentra of arterial and venous blood to equilibrate Anesthetic uptake is influenced by many of the same factors that influence transfer from lung to blood: tissue-blood partition ‘oelicients, nites of blood fl to the tissues, and concentration gradients are all important factors. Highly perfused tiseues (brain, heart, liver, kidneys, and splanehnie bed) will exert the greatest inf teriovenous concentration Skin and muscle undengo slow diffasion because these tissues are less richly perfused and thus exert less ofan effect ‘on arteriovenous concentration "The mechanism isnot clear. All anesthetics have the common property of increasing the threshold of action ‘potentials and inhibiting the rapid in membrane permeability to mons. The effect ofthese changes own, No receptors have been found to interact with general anesthetics, isnot Halothane Enfurane Isofarane Desflurane 5, Sevoflurane 6. Nitrous oxide Using the concept of MAC ‘What is MAC? How docs MAC relate to poteney? Flow ean the MAC of any inhaled agent be reduced? Halothane Describe this drug. ‘What is its clinical indication? Is this deug metabolized? ‘What is halothane’s MAC? What are the cardiovascular effects of halothane? Are there Choptor 16/ Anesthetics 105, MAGis the minimum alseolar concentration of an ancsthetie necessary tocliminate movement among 50% of ptients challenged by a standardized skin ‘The greater the MAC of an agent; th greater the concentration needed to provide anesthesia, Thus, an agent with a high MAC has alow poteney (for ‘example, nitrous oxide), By using the agent in conjunction with analgesics such as opioids or sedative hypnotics ‘The first of the halogenated volatile anestheties to he developed, halothane as been largely replaced by more wdern agents [tis still used in the podiatiie population Ihecause of ts pleasant odor and lack of hepatotoxicity Approximately 20% is eliminated by metabolism; the remainder is eliminated unchanged in expined air Halothane sensitizes the myocardinn to the effets of eatecholamines (thus Increasing the risk of arrhythmia), decreases heart rate and cardiae output, and leads to lowered BP and peripheral ‘epatotoxicity Halothane hepatitis, filminant hepatic necrosis—has been ‘associated with halothane, althongh halothane has not been directly plicated us the cause, Malignant hypertherinia Halothane can induce malignant hyperther secondary tits absorption andl metabolism in skeletal musee, 106 Section il / Central Nervous System What is mali thermia? want hyper- How is this condition treated? A potentially fatal reaction to any ofthe Inhaled anesthetics, which results in hyperthermia, metaboli acidosis, tachycardia, and aceclerated mm contraction le ‘With dantrolene and cessation ofthe offending agent Is this drug metabolized? ‘What is enflurane’s MAC? What are this drug’ eardio- vascular effects? Describe the toxic effets. Isoflurane What are the clinical indications? Is this drug metabolized? What is its MAC? What a effects? the cardiovascular Rapid induetion of general anesthesia Approximately 2% of the agent is metabolized to a fluoride ion, which is then exereted by the kidney. The rest is climinatod unehanged in the expired air. Les Enflurane is similar to halothane in that it decreases heart rate, BP, and peripheral resistance, Although it has less of sensitizing effect on the myoeardiuin than does halothane, enflurane can nevertheless produce cardiac arrhythmia ‘The fluoride ion resulting from cenfranc’s metabolism can be nephrotoxic, A decrease in the kidney's ‘concentrating ability after prolonged ‘exposure to enilarane has been observed in patients with preoperative kidney insufficiency. ‘General anesthesia Isoflurane is minimally metabolized, almost all of the drag is eli tmehanged i the expined air 14% Does not effect eurdiae output Chapter 16 Anesdetcs 107 Can lower BP and reduce peripheral vascular resistance profoundly ‘Does not sensitize the myocardium to catecholamines oes not induce arrhythmia Are there any toxic effects? Potential for lignant hyperthermia Desflurane Deseribe this drug's clinical General anesthesia indications. Does this drug undergo Desflurane is eliminated unchanged in ‘metabolism? the expired aie, ‘What is its MAC? 6% What are desflurane’s Its profile very similar to that of cardiovascular effects? isoflurane Are there toxic effects? Potential for malignant hyperthermia ‘Sevoflurane What are the clinical General anesthesia indications? Does sevoflurane undergo A stall percentage of this agent is metabolism? ‘metaholized to foride ion; the remainder is eliminated unchanged inthe expired sir What is this drug’s MAC? 25 Deseribe sevoflurane’s thas acardiae profile similar to that of cardiovascular effects. Doth desflurane and isoflurane. ‘Ave there any torle effects? Although metabolism of sevollurane Fees loshin om ale yepalfreton lke enfirme dvs thas been pod sevallrin is nt metabolised nthe Kidoey (or eflrae ox effects do not our ied that bees Nitrous Oxide (“Laughing Gas") What are the clinical Induction of general anesthesia 108 Secon il Central Nervous System How is this drug Because ofits low potency, nitrous oxide administered? must be used in conjunction with other anesthetics (either inhaled or intravenous) for effective anesthesia. Tsnitrous oxide metabolized? No What is its MAC? 100% —Even if 100% nitrous oxide were given to patients, surgical anesthesia would not be achieved (and profound hypoxia would result) Are there cardiovascular Nitrous oxide minimally allets the effects? cardiovascular system, Name a major contra nitrous oxide is administered to indication to the use of patients who have closed air cavities (6, nitrous oxide. pneumothorax), the gas will diffuse into the cavity and increase the pressure ‘within it What special considerations toadequatel should be taken into account oxyzenate the patient, becaasenitrons during the recovery phase? oxide will difise from the blood into the alveoli so quickly that i entirely replaces oxygen, which results in a diffusion hypoxia. What are the toxie effects? Nitrous oxide ean cause bone marrow depression with prolonged administra tion: high concentrations may cause: neuropathies INTRAVENOUS AGENTS What are the classes of Barbiturates, benzodiazepines, opioids, intravenous anesthetics? and dissociative agents Ultra-Short-Acting Barbiturates Thiopental ‘What is the clinical indi- ‘Thiopenta is used for the induction of cation for this drug? snesthesia in combination with inhaled anesthetics, Ithas « rapid onset of action ‘with unconseiousness occurring 15 to 30 seconds alter acinistration, What is this drug's mecha- nism of action? What phenomenon responsible for this drug’s fast onset of action and. short duration? 1s thiopental metabolized? What are the cardiovascular effects? Deseribe the respiratory effects, What are this drug’s effects ‘on cerebral blood flow? Aro there any side effects? Chapter 16 Anestheues 1097 “Thiopental binds to the y-aminobutyrie acid A (GABA,) reoeptor, which results {in prolonged opening o chinmel. The nengon’s membrane is thus hyperpolarized, which creates a reduction ‘of neuronal excitability lorie “Thiopental has high lipid solubility and thus exosses the blood-brain barrier very ‘quickly. However, the drug jast as quickly diffuses out of the brain and other highly lis rapidly ‘vascularized organs, realistributed to muscle body tissues, which accounts for its short Yes. Mi much more slow redistribution, Nearly 90% of the drug is metabolized; thus, after lange doses (especially continuous infusions), recovery can be very slow. TThiopental reduces blood pressure and cardiac output, but it does not affect peripheral resistance, ‘Thiopental depresses the respiratory center ofthe brain and blunts the response to CO, and hypoxia. This effect ‘on cerebral blood flow makes thiopental a desirable agent in patients who have cerebral edema, It decreases cerebral blood flow and ‘oxygon consumption by the brain, ‘Thiopental may induce laryngospasin and bronchospasm. It may also exacerbate cate intermittent porphyria by inducing, the synthesis of hepatic 8-aminoleyalinie acid synthase (thiopental has precipitated porphyric crisis) Benzodiazepines—Midazolam, Diazepam, Lorazepam See Chapter 11—Aniolyties, Hypnoties,and Sedatices for further information ‘on the benzodiazepines NO Section Central Nervous System Deseribe the clinical indi-__Benzovliazepies are used for cation for these drugs. preoperative sedation, intraoperative sedlation for procedures not requiring analgesia (colonoscopy, cardioversion, and so forth), and as part of balanced anesthesia (using several agents simultaneously to obtain surgical anesthesia), What isthe mechanism of —Benznadiazepines hind GABA receptors, action? ‘which results in reduced neuronal ‘excitability, These drugs have a much slower onset of aetion than do the Darbiturates. Why is midazolam used asa ‘This drug produces antegrade amnesia pre-anesthetic medication? (loss of memory of events alter administration of the drug), which calms the patient Why is midazolam also the Midazolam has a shorter onset of action, preferred benzodiazepine greater potency, and mote rapid for induction and mainten-_limination when compared to other ‘ance of anesthesia? benzodiazepines What are the side effects of Aono, these drugs can cause moderate benzodiazepines? circulatory and respiratory depression, If used with opioids, cardiovascular collapse and respiratory arest can oovur, Are there other uses for ‘They are used to contr sefznzes and in these drugs? the aleohol withdrassal process ta control symptoms, Isthere abenzodiazepine -Flumazenil antagonizes the CNS antagonist? depression caused by benzodiazepines Opioids—Fentanyl, Morphine What are the clinical indi-_Opivids are used as general anesthetics in cations for these drugs? patients undergoing cardiae surgery or ‘other major surgeries for which curdive reserve is limited, Why is fentanyl used more Tthasa greater potency and less impact often than morphine? ‘on respiratory drive than dees morphine Are there any side effects? Is there an opioid antagonist? What i Innovar? Other Agents Propofol (Diprivan) Describe this drug's clinical indications. ‘What are the pharmacologic characteristics of this drug? ‘Why is propofol preferred ‘over thiopental for induction. of anesthesia? How is propofol metabolized? What are the side effects? Chapter 16/Anesthenes 111 TV opioids can cause chest wall rigidity, ‘which makes ventilation more difficult. Postoperative respi story depression ean Intraoperative awareness with unpleasant postoperative recall may occur. Naloxone reverses the respiratory and ONS effects of opioids Tnnowar isa combination of fentanyl and droperidol, When used with nitrous oxide, Innovar produces neurolept- anesthesia (combined analgesia snd Induction of anesthesia Ulrafast-acting drug with pharmacodynamics similar to that of thiopental High lipid solubility ery quick distribution to highly vaseularized sites (e.g. the br Rapid diffusion hack into the blood subsequent redistribution with Rate of onset very similar to thiopental Recovery’ is quicker, with patients able to ambiilate sooner than with other 1 anestheties| Patients report less nausea and emesis ‘No cumulative effeet from propofol administration or delayed recovery ater prolonged infusion Its rapidly metabolized by liver and other extrahepatic enzymes (10 times ‘more quickly than is thiopental. Hypotensio Neqative inotropic effects Pain atthe site oFinjction (most ly enconntered side effect) 112. Secon tl Central Nervous System Does propofol have any ther uses? Ketamine Deseribe the elinieal indications. What is the mechanism of action? What are the cardiovascular effects? Is ketamine used in head trauma cases? What are the side effects? What can be done to reduce the incidence of this phenomenon? Are there other routes of ration? adi LOCAL ANESTHETICS Propofol is often used! in the critical care setting asa continuous infusion to provide prolanged sedation because ofits unique cardio vascular effets s used in trauma eases where cardiowaseular support is necessary. Its also used in children undergoing painful procedures (dressing. changes of bums) orto facilitate coop. daring radiographic procedures nine produces a dissociative anesthesia characterized by catatonia, and analgesia without actual loss of consciousness Ketamine is unique in that it produces cardiovascular stimulation, with heart arterial blood pressure, and eardiac coutpat all usually This drug stimulates the sympathetic nervous astern, which cases the release cantly increased, of catecholamines, Na, because it inereases cerebral blood flow, oxygen consumption, and {intracranial pressure, Ket phenomenon consisting of disorientation sensory and perceptual ilasions, and Vivid, often unple nine ean cause an emergence u Ketamine adiinistration ‘of diazepam 5 to 10 minutes before Yes, Ketamine can be given intramus: ccilarly as well as intravenously What are the two types of local anesthetics? ‘The two classes are determined by the bond linking the lipophilic portion of the Name the ester anesthetics. Name the amide anesthetics. anesthetics differ? What is the mechanism of action of local anesthetics? Which nerve fibers are most sensitive to local anesthetics? What are the elinieal indi- cations? How is the duration of action of local anestheties increased? Are there any side effects? Chapter 16 Anesthercs 113 molecule with the hydrophilic ‘component—cither an ester or amide bond. Cocaine Benaocaine Procaine Tetrweaine Bupivacaine Prilocaine Esters are mote rapidly metabolized by blood and tissue estemises, which gives them shorter half-lives. Amides ae metabolized by hepatic microsomal Life res, which resus in a longer hal wuction by These agents block nerve ¢ inhibiting the voltage-gated soa channels of the-nesstonal membune Small, unmyelinated fibers that conduct pain, temperate, and autonomic activity ane alfected first, With increasing concen trations of anesthetic. p d affected then sensory fibers (A fibers) an last, motur newrons (A fibers) Bers ( A fibers) ar Local anestheties are used for surfiee aanesthesta, nerve blacks, and spinal andl epidural anesthesia, Lidocine is also a potent anti-arshythmic Adin reduces blood flow tothe anesthetized. ws the absorption of ic their duration of phrine to local esthetics area, thus deere the drugs and en Systemic effects of local anesthetics occur with high doses of the drug ONS disturbances—Lightheadedness, 114 Section i / Contal Nervous Sytem sensory disturbances, convulsi ‘coma, and even death at high doses Cardiovaseular effects —Myoeadial depression and hypotension, except for cocaine, which causes ‘vasoconstriction and hypertension METHYLXANTHINES CNS Stimulants ACNS stimulant isa drug that increases niotor activity, enuses excitement, and decreases feelings of fatigue. CNS stimulants include the methylsanthines, ne, and the amphetamines What are methyleanthines? How do methylxanthines work? CAFFEINE What are the physiologic effects of caffeine? A gronp of psychomotor stimulants including Calfeine ‘Theophylline ‘Theobromine (found in cocoa but of litle ‘Research indicates that methylsant nerease cyclic guanosine monophosphate (cGMP) and eyclic adenosine monophos phate (cAMP) by inhibiting phospho- diesterase and blocking adenosine receptors Calfeine effects a number of organ systems within the body: CNS—Calfeine increases motor activity ‘and alertness, Cardiovaseular—Caffeine increases heart rate and contractility, ‘Smooth musele—Caffeine and its derivatives relax the smooth 1a ‘af the bronchioles. Genitourinary—Caffeine can act as weal diuretic and increase output of Na*, CI, and K Gastrointestinal —Caffeine stinulates us Secton I Central Nervous System secretion from the gastric mucosa Therefore, patients who have peptic uleer disease should be eounseled to avoid caffeine. What are the adverse effects At low doses Insomnia and agitation ean of chronie eaffeine use? occur. At highher doses (8-10 11 convulsions, ancl even cardie Do methylxanthines cross Yes, and they are seereted into the the placenta? mother's milk, Patients should be advised to avoid them during pregnaney and while nursing. THEOPHYLLINE See Chapter 28—Drugs Used to Treat Asthina, Coughs, and Colds, for adie tional discussion of theophylline What is the therapeutic role ‘Theophylline ean be used in the of theophylline? treatment of asthma, but currently itis not being used frequently because it has a very narrow therapeutic index and is not aselfective as the new B agonists, NICOTINE How are the physiological In low doses, nicotine causes ganglionic effects of nicotine related stimulation by depolarization. At high to the dose? doses it causes ganglionic blockade, What are the physiologic At low doses —arousal, relaxation, and actions of nicotine on the improved attention ‘central nervous system? Athigh doses—central y paralysis eaused by disruption of | ‘medullary Function How does nicotine affect At low doses increase in blood pressure the peripheral nervous heart rate: constriction of blood system? vessels to the digits and impairment of flow At high doses—deerease in blood pressure and in action of GL and GU tract due to ganglionic blockade What are nicotine’s Nicotine has no therspentic uses, therapeutic uses? What is nicotine’s route of administration? What are its adverse effects? What withdrawal symptoms do nicotine addicts experience? AMPHETAMINES. (Chapeer 17 CNSStimulanes 117 Absorption oceurs through oral mucosa, by inhalation, and transdlermally ENS—invitablity and Poripheral—intestinal eramps, diarrhea, and inereased heart rate and blood pressure A crwing for tobacco Is nccompanied by invtability, restlessness, anxiety, and gastrointestinal pain Name three drug class. ples of this How do the c drugs work? What are the physiologic actions of these drugs? What is their clinical use? What is the route of admin- istration? Where are amphetamines metabolized? Does physiologic and psy- chological dependence ‘occur with amphetamine use? What are the adverse effects of these drugs? 1, Methylphenidate (Ritalin) Methamphetamine (Methedrine)— speed” 3, Destroamphetamine (Dexedrine) Amphetamines work by releasing neuronal stores ofeatecholamines, ‘especially norepinephrine and dopamine Exphoria Decrease in fatigue Imerease in blood pressure Increase in ate af respirato Decrease in appetite Attention deficit hyperactivity disorder (ADITD)—MethyIphenidate is used to alleviate this problem, Appetite control—Amphetamines decrease appetite by blocking the receptors in the kateral hypothalamus Nareolepsy Onl In the fixer Yes—umphetamines ean be very addictive Amphetamines, ike caffeine and nicotine, affeet multiple organ systems:

You might also like